CASE SCENARIOS

Case Scenario 1

Background : 39-year-old bartender presents to her GP with a four-month history of being generally unwell, loss of appetite, abdominal discomfort and increased bowel movements. She also states that she passes air occasionally when she attempts to pass urine. Her menstrual periods are regular.

Social History : She smokes about 20 cigarettes per day and drinks 10units of alcohol per week. Examinations: Temperature = 39.8°C and she is tender over her right iliac fossa. Per rectal examination is unremarkable but a few small ulcers are noticed around the perianal region.

Investigations :
Bloods : White cell count = 14.4 (high) C-reactive protein level = 98 (high) Haemoglobin = 10.5 MCV: 80
Colonoscopy : Skip lesions of linear ulcers and transverse fissures giving cobblestone appearance of the mucosa.
CT scan : Reveals enterovesical fistula

Case Scenario 1

On review of the patient consider what is the most likely diagnosis and possible differentials?

Consider the potential differentials above and work your way through the content to help revise the clinically relevant conditions before confirming your answer.

Pelvic Inflammatory Disease Definition: Generalised term relating to infection of any part of the upper female genital tract – this compromises of the following structures:
  • Uterus
  • Fallopian tubes
  • Ovaries

Causes : PID if often associated with sexually transmitted infections (Gonnorhoea and Chlamydia) whereby the organism ascends from the cervix and can cause significant damage and long term complications. Long term complications of untreated PID include infertility, ↑risk ectopics, abscess formation and chronic pelvic pain.

Pelvic Inflammatory Disease
Risk factors:
  1. STI bacterium most common – full Hx for sexual activity required
  2. Previous episodes
Presentation:
  1. Lower abdominal/suprapubic pain
  2. Fever (temp >37.5◦C)
  3. Pain during sexual intercourse
  4. Irregular menstrual cycle +/- dysuria
  5. Vaginal or cervical mucopurulent discharge
Positive swab test showing mucopurulent discharge
Complications:
  1. Ectopic pregnancy - tubal damage impacts the ability for normal implantation increasing the risk of ectopics
  2. Infertility - symptomatic and asymptomatic PID can cause permanent injury to the fallopian tube impacted the cilia and reproductive potential
  3. Chronic pelvic pain – PID can cause pelvic pain that may last for months or years
  4. Ovarian Cancer – Associated with multiple episodes of PID likely due to the increased incidence of nulliparity, nulligravidity, and infertility (known) risk factors for ovarian cancer
Pelvic Anatomy – Upper Female Genital Tract

Objective – Match the below anatomical features and think of the common areas that would be impacted in the pathology of PID. Consider the route of infection and where the ascending bacterium travel and equally brainstorm the likely risk factors implicated in PID


Carcinoma of the Caecum

Localised carcinoma of the caecum (originating from adenoma) often regarded as a right sided colorectal tumour which would normally also include ascending and proximal transverse colon.




Risk Factors
  • Increases with age – mostly >60y
  • Familial component (FHx)
  • Inflammatory disease e.g. IBD, diabetes (increased cell turnover)
  • Poor diet – high in fat, low fibre (generally for all colonic cancers)
  • Obesity
  • Smoking
Presentation
  • Unintentional weight loss over short period (red flag)
  • Unexplained pain in the right iliac fossa – late sign of local tumour
  • Change in bowel habit
  • Malaise and weakness secondary to underlying anaemia caused by unrecognised blood loss
Carcinoma of the Caecum Select the correct anatomical position of the caecum






Note: Pathology of the caecum would tend to cause RIF pain as seen in the scenario BUT be aware of other causes of pain in this region. Note also for other clinical signs and symptoms to help exclude this diagnosis

Crohn's Disease

Definition : Crohn's disease is a chronic relapsing and remitting inflammatory condition affecting any part of the GI tract from the mouth to the anus. It is characterised by transmural inflammation with non-caseating granulomas and on endoscope a 'cobblestone' appearance can be observed. The disease can affect any part of the gastrointestinal tract from the mouth to the anus, although it frequently affects the terminal ileum (terminal ileitis). Inflammation occurs in all layers, down to the serosa increasing the risk of complications such as strictures, fistulas and adhesions


Risk Factors : The precise aetiology is unknown but an altered response by the body’s immune system to normal intestinal bacteria are implicated. Factors therefore include:

  • Smoking (smokers have a x3-4 ↑ risk vs non-smokers)
  • Environmental factors
  • Familial predisposition
  • Theories linking pathogenic bacteria and viruses has all been implicated

Crohn's Disease

Patient’s presenting with a symptomatic flare up of Crohn’s disease will display symptoms relating to acute inflammation. This can cause associated pain & discomfort, change in bowel habits, decreased absorption, weight loss and anaemia. The inflammatory process can affect all layers of the gut, down to the serosa. This is why patients with Crohn's are prone to other complications e.g. strictures, fistulas and adhesions


Revise the layers of the small intestine. Complete the gaps by matching the correct anatomical definitions:



Crohn's Disease

Presentation:

  • Non bloody diarrhoea with ↑ frequency/urgency
  • Tenesmus
  • Abdominal pain/tenderness – dependant on where inflammation occurs
  • Weight loss/malaise/fever
  • Perianal skin tags, abscess
  • Fistula and strictures may be present

Investigations:

  • Bloods: FBC, CRP, ESR, Ferritin
  • Stool culturing – checking for common pathogens e.g. E.Coli, C.Difficile, Campylopacter
  • Colonoscopy +/- rectal biopsy
  • MRI – can assess pelvic disease and locate strictures/fistula

Colonoscopy showing continuous "cobblestone" appearance of the colon
arrow_forward
Crohn's Disease - Most commonly affected areas

Inflammatory lesions in:

  • 15%–25% of patients are limited to the colon
  • 40%–55% are seen in the in the terminal ileum and the colon
  • 25%–40% are exclusively in the ileum
  • 1%–10% involves the oesophagus, stomach, and proximal parts of the small bowel

Patients can often also present with other symptoms related to Crohn’s disease which do not directly effect the GI tract. These are called extra-intestinal signs/symptoms. These include:

  • Complications of the big joints
  • Eye problems – episcleritis, iritis
  • Dermatology conditions e.g. erythema nodosum

Crohn's Disease - SBA
Define what a stricture is?

Define what a fistula is?

Ulcerative Colitis

Definition: Ulcerative colitis (UC) is a form of inflammatory bowel disease (IBD). It is a relapsing and remitting condition affecting the the colonic mucosa only. Inflammation always starts at rectum (hence it is the most common site for UC), never spreads beyond ileocaecal valve and is continuous. 2 key peaks in incidence are observed in UC therefore be suspicious of the 2 age groups when presenting with abdominal symptoms: 15-25 years and those aged 55-65 years

Risk Factors: The precise aetiology is unknown but there are some known factors associated with UC

  • Age – highest in 15-25y age group and then 55-65y age group Family Hx – if having a parent of sibling with the disease Ethnicity/Race – White have a higher prevalence and especially Ashkenazi Jewish descent Smoking status – UC is 3 fold more common in non-smokers (smoking may have a ‘protective role’ but is still not advised
Ulcerative Colitis - Imaging

Complications:

  • Toxic dilatation – excessive widening of the colon >6cm known as Toxic Megacolon – decreasing motility, thinning of the wall and loss of haustra markings as seen on the X-Ray <
  • Haemorrhage – severe rectal bleeding uncommon but can occur <
  • Perforation – grave complication of UC, this can occur without dilatation and has a 40% mortality rate<
  • Colon Cancer – increased long term risk <
Note : Ulcerative Colitis can be assessed and categorised into
  • Mild
  • Moderate
  • Severe
This accounts for various factors e.g. bleeding, frequency, observations and blood tests – Truelove and Witts Criteria
Differentiating between Crohns disease vs UC

Many of the presentations in IBD acutely can appear quite similar. It is therefore useful to know what signs and symptoms overlap and others that favour one diagnosis over the other – see below.

This will factor in taking a detailed history from the patient but also doing a thorough and relevant examination accounting for the abdominal exam, assessment of the perianal region and assessing for extraintestinal signs.

Coeliac Disease

Definition: Coeliac disease is an autoimmune disease (T-cell mediated) caused by sensitivity to the protein gluten. Individuals affected by coeliac are intolerant to specific gluten proteins found in wheat, barley, rye +/- oats. Repeated exposure to the specific protein causes inflammation and villous atrophy in the small intestine which in turn causes malabsorption. Conditions associated with coeliac disease include dermatitis herpetiformis (a vesicular, pruritic skin eruption) and autoimmune disorders (type 1 diabetes mellitus and autoimmune hepatitis). It is strongly associated with certain genotypes e.g. HLA-DQ2 (95% of patients) and HLA-B8. It commonly presents in infancy as faltering growth but equally peaks again in 50-60y olds.

Presentation: Note 1/3 of patients are asymptomatic

  • Most common in infancy = faltering growth, ++diarrhoea and ongoing fatigue and signs of anaemia ‘lack lustre child’
  • Bowel changes - often mistaken for irritable bowel syndrome (IBS) , malabsorption and anaemic symptoms most common in 50-60y age group
  • Smelly stools and steatorrhoea (difficult to flush)
  • Abdominal symptoms – bloating, cramping with nausea +/- vomiting
  • Signs of nutritional deficiencies – calcium = osteomalacia and B vitamins = anaemia, angular stomatitis
Coeliac Disease

Definition: Coeliac disease is an autoimmune disease (T-cell mediated) caused by sensitivity to the protein gluten. Individuals affected by coeliac are intolerant to specific gluten proteins found in wheat, barley, rye +/- oats. Repeated exposure to the specific protein causes inflammation and villous atrophy in the small intestine which in turn causes malabsorption. Conditions associated with coeliac disease include dermatitis herpetiformis (a vesicular, pruritic skin eruption) and autoimmune disorders (type 1 diabetes mellitus and autoimmune hepatitis). It is strongly associated with certain genotypes e.g. HLA-DQ2 (95% of patients) and HLA-B8. It commonly presents in infancy as faltering growth but equally peaks again in 50-60y olds.

Investigations:

  • Blood tests – FBC e.g. Haemoglobin (Hb)
  • Anaemia screen e.g. Vit B12, Ferritin levels (usually microcytic MCV < 70fL)
  • Antibodies: Tissue transglutaminase, anti-endomysial (both specific for coeliac to determine gluten intolerance)
  • Endoscopy for duodenal or jejunal biopsy to assess for:
    • Villous atrophy
    • Crypt hyperplasia
    • Lymphocyte infiltration
Note :

For the above testing to be confirmation of diagnosis the patient must remain on a gluten containing diet to ensure a valid result. Therefore always ask in the history about diet and what if anything has been excluded and for how long.

Think back to case 1. Accounting for the highlighted features within the history and investigations a probable diagnosis is?

Background : 39-year-old bartender presents to her GP with a four-month history of being generally unwell, loss of appetite, abdominal discomfort and increased bowel movements. She also states that she passes air occasionally when she attempts to pass urine. Her menstrual periods are regular.

Social History : She smokes about 20 cigarettes per day and drinks 10units of alcohol per week.
Examinations: Temperature = 39.8°C and she is tender over her right iliac fossa. Per rectal examination is unremarkable but a few small ulcers are noticed around the perianal region.

Investigations :
Bloods : White cell count = 14.4 (high) C-reactive protein level = 98 (high) Haemoglobin = 10.5 MCV: 80
Colonoscopy : Skip lesions of linear ulcers and transverse fissures giving cobblestone appearance of the mucosa.
CT scan : Reveals enterovesical fistula

Case Scenario 1

Case scenario 1: Explanation

The patient is presenting with common clinical features of Crohn’s disease. 

The most common presentation of patients with Crohn’s disease is related to the long term inflammatory process involving the ileo-caecal region which includes low-grade fever, loss of appetite, weight loss (anorexia), anaemia  and general malaise.

Patients may have crampy or constant abdominal pain but often on examination this will be over the umbilical region or over the right iliac fossa. 

Pain may be relieved by defecation but is usually temporary relief. Diarrhea may be troublesome, which is usually non-bloody and intermittent.  If the colon is involved, patients may present with diffuse abdominal pain accompanied by mucus, blood and pus in the stool which are features normally associated with ulcerative colitis. 

Always consider the extra intestinal manifestations of Crohn’s disease assessing for skin rashes, erythema nodosum, arthritis and inflammation of the eye such as uveitis – although none applied in this case.  On endoscopic examination, crohn’s disease is characterised by skip lesions (patchy areas of inflammation) and described as cobblestone appearance.  Some important complications of Crohn’s disease include fistula formation (e.g., entero-colic, entero-cutaneous), abscesses (e.g., perianal), haemorrhage, and strictures causing intestinal obstruction.  This patient has developed an enterovesical fistula which causes her to pass air when she attempts to pass urine. 

The management of Crohn’s disease depends on the severity and the stage of the disease.  The treatment may be conservative (including medical) or surgical treatment. This is aimed at reducing inflammation/infection and removing damaged areas of the gut.

For further information for patients and your own revision it is worth visiting the Crohn’s and Colitis charity webpage  www.crohnsandcolitis.org.uk

Case Scenario 2

Background : A 78-year-old gentleman presents to his GP with a three-month history of non-specific lower abdominal discomfort, abdominal distension and constipation. On further questioning he says that he has noticed a change in his bowel habits, a sense of incomplete evacuation of the bowel and his stools to be mixed with blood.

Social History : He lives alone and is an ex-smoker and drinks 10units of alcohol per week. He reports a good appetite but states his trousers have become loose fitting in the last few months

Examinations : Temperature = 37.2°C, normal heart rate. Abdominal palpation is soft and non-tender but you notice a mass at the left iliac fossa. Per rectal examination is uncomfortable but not painful and your glove shows some spotting of blood and mucus

Investigations :
Bloods : U&Es: normal, Haemoglobin = 11.5 (L), MCV: 70 (L), White cell count = 9.5 (normal), C-reactive protein < 5 (normal)
Imaging : Plain abdominal X-ray reveals a dilated descending and transverse colon.

Case Scenario 2

On review of the patient consider what is the most likely diagnosis and possible differentials?

Consider the potential differentials above and work your way through the content to help revise the clinically relevant conditions before confirming your answer.
Toxic Megacolon

Definition : A complication of IBD – more commonly ulcerative colitis or occasionally in the older population caused by clostridium difficile infection. This is an acute presentation and often a medical emergency. Characterised by excessive colonic dilatation transverse colon usually greater than

(Select the correct figure >4cm/>5cm/>6cm)




This can cause severe complications:

  • Perforation of the colon
  • Sepsis
  • Shock

Clinical presentation – Select the most likely findings below

Note : Typical history e.g. 25-year-old man presenting with abdominal pain, passage of blood and mucus per rectum, abdominal bloating, fever, and disorientation – worsening acutely over the last 2 days. Surgical intervention is often required as treatment
Diverticular disease

Definition: Diverticular disease is a common surgical problem. It consists of herniation of colonic mucosa through the muscular wall of the colon – causing outpouching or sacs to form in the lining of the colon. This occurs in areas of weakness along the lining and where pressure in the colon is highest. A single out-pouch or sac is known as a diverticulum other important terms to be familiar with include


Diverticulosis – multiple diverticulum are present BUT asymptomatic
Diverticular disease – multiple diverticulum present BUT symptomatic
Diverticulitis – acute flare up of a diverticulum becoming symptomatic requiring treatment

Clinical presentation

  • Altered bowel habits
  • Abdomen pain
  • Bleeding
  • Nausea
  • Increased flatulence
  • Pain relieved by defecation

Note : Diverticulum more commonly form near penetrating vessels along the overlapping longitudinal muscle layers (taenia coli) and in the sigmoid colon as there is highest level of pressure present here.
Diverticular disease

Risk Factors: We understand that the pathology relates to increased intraluminal pressure and areas of weakness along the colonic mucosa hence why the sigmoid colon is most commonly affected (95% of complications occur here). Now consider potential factors which may influence this


  • Age – increasing age leads to weakened and less pliable colonic mucosa
  • Dietary factors – low fibre intake (especially soluble fibre)
  • Decreased gut motility – drug related or chronic constipation
  • Obesity
  • Hiatus hernia can increase pressure

Complications: It is believed 30% of over 60s have some extent of diverticular disease but are often asymptomatic. Of those who develop more severe diverticulitis 25% are likely to have complications which can include:

  • Abscess formation - pus collects in the pouch
  • Obstruction/blockage of the colon caused by scarring
  • Fistula formation
  • Perforation – infected/inflamed pouch ruptures, spilling intestinal contents into the abdominal cavity causing peritonitis (medical emergency requiring immediate care)
Sigmoid volvulus

Definition:Sigmoid volvulus is a cause of large bowel obstruction and occurs when the sigmoid colon twists on the sigmoid mesocolon. Often when twisting this causes a closed loop of bowel filled with faeces and gas and becomes enlarged. This is the cause of 1/3 of large bowel obstructions in adult but is rare in children.


Clinical presentation

  • Constipation or bowels not opening
  • Failure to pass flatus
  • Abdominal bloating/distension
  • Colicky abdominal pain
  • Nausea/vomiting
Note : On examination there is often a palpable, non-tender mass. Per rectal examination highlights an empty rectal ampulla. Abnormal observations e.g. increased HR, temperature >37.5°C and hypotension are a sign of shock and indicate perforation.
Sigmoid volvulus

Risk Factors and Associations:

  • The elderly
  • More common in males
  • Chronic constipation
  • Neurological conditions e.g. Parkinson's disease, Duchenne muscular dystrophy
  • Psychiatric conditions e.g. schizophrenia
Diagnosis
  • Plain abdominal X-ray: Large bowel obstruction showing dilated sigmoid loop with a double line in the middle made of the medial walls – often called the 'coffee bean' sign
  • CT scanning is the least invasive imaging technique that allows assessment of bowel wall ischaemia.
Note :Differential diagnosis for this presentation may also include carcinoma of the sigmoid colon, severe constipation and giant sigmoid diverticulum therefore through medical history is required to help distinguish
Acute colonic pseudo-obstruction

Definition:Acute colonic pseudo-obstruction (ACPO) also known as Ogilvie's syndrome is a disorder characterized by acute dilatation of the colon in the absence of an anatomic lesion that obstructs the flow of intestinal contents. The term pseudo is in reference to the signs and symptoms presence of obstruction but in the absence of a mechanical blockage. ACPO is commonly associated with severe illness or post-operative complications often in conjunction with a metabolic imbalance.

Risk Factors:

  • Trauma
  • Pelvic, abdominal or cardiothoracic surgery
  • Severe illness e.g. M.I, Heart Faiure
  • Retroperitoneal pathology e.g. malignancy

Medications which ↑ Risk

  • Phenothiazines
  • Opiods
  • Alpha-2-adrenergic agonists
  • Methotrexate

Diagnosis: The diagnosis of acute intestinal pseudo-obstruction should be suspected in patients with abdominal distension or pain and a abdominal examination which reveals a distended and tympanitic abdomen. The diagnosis of acute intestinal pseudo-obstruction is established by abdominal imaging – CT +/- contrast.

Acute colonic pseudo-obstruction

Clinical Features:

  • Abdominal distension – occurring over 3-5 days
  • Abdominal pain
  • Abdominal exam – tympanitic but bowel sounds present
  • Nausea and vomiting
  • Constipation or Diarrhoea

Signs indicating severe disease, colonic ischaemia or perforation:

  • Fever
  • Marked abdominal tenderness
  • Signs of peritonitis e.g. guarding, rigidity, rebound tenderness

Imaging: The presence of dilatation of the large bowel shows haustra present and loops located peripherally. Commonly there is NO evidence of an abrupt transition point or mechanically obstructing lesion.

Colonic Carcinoma

Definition: About two thirds of all colorectal tumours develop in the colon and the remainder in

Risk Factors:

  • Family history of carcinoma : adenoma under the age of 60 years..
  • IBD : ulcerative colitis, Crohn's colitis
  • Previous history of small bowel cancer, endometrial cancer, breast and ovarian cancer
  • Syndromes/conditions predisposing an individual to increased polyps often familial and detected early
  • Hereditary non-polyposis colorectal cancer (HNPCC)
  • Hormonal factors : no pregnancies, late age at first pregnancy, early menopause
  • Diet : low fibre intake and increased red meat
  • Sedentary lifestyle, obesity, smoking, high alcohol intake

Colonic Carcinoma

Colonic Polyps: Small non-cancerous (benign) growth of cells form on the inside lining of the colon or rectum. Often consisting of a short stalk and a head likened to a mushroom in shape which protrudes from the internal lining of the bowel. Some people develop just one bowel polyp but it is not uncommon to develop two or more. These most commonly develop in the descending colon but can occur anywhere.

Progression to Colonic Carcinoma:

Most colorectal cancers develop from a polyp that has been present for 5-15 years. Abnormal cell growth can be triggered due to various factors which may damage the DNA of the cells causing cancerous changes.

Colonic Carcinoma

Common locations: See on the right hand diagram for commonly quoted statistics of incidence. Its has also been observed that black females tend to have a more proximal presentation (right sided) vs male Caucasian who tend to have a more distal presentation (left sided).

Presentation: See on the right hand diagram for commonly quoted statistics of incidence. Its has also been observed that black females tend to have a more proximal presentation (right sided) vs male Caucasian who tend to have a more distal presentation (left sided).

Progression to Colonic Carcinoma:

Often signs and symptoms can relate to the location of the tumour and are often described as being ‘Left sided’ or ‘Right sided’ although this should be seen as a guide vs a definitive criteria.


Left sided Right sided
Colicky pain and early change in bowel habit Weight loss
Rectal bleeding Anaemia - ↓Hb
Tenesmus Occult bleeding
Mass in left iliac fossa Mass in right iliac fossa
Bowel obstruction more common Obstruction less likely
Case Scenario 2

On review of the patient consider what is the most likely diagnosis and possible differentials?

Case scenario 2: Explanation

Elderly patients with per rectal bleeding, change in bowel habits, and weight loss are all deemed as ‘’red flags signs’ should be considered to have colonic cancer unless proven otherwise.

Increase in age is a risk factor for colonic cancer. The other risk factors include, a family history of colon cancer, familial adenomatous polyposis, diet rich in red meat/low fibre and long standing ulcerative colitis or Crohn's colitis.

The clinical presentation of patients with colonic malignancy depends on the site of the tumour:

  • Right-sided colonic carcinoma commonly presents with anaemia, tiredness, malaise, pallor and loss of weight.
  • Left sided colonic carcinoma presents with change in the bowel habits, bleeding per rectum and intestinal obstruction.
  • Rectal carcinoma, in addition to the features seen in left-sided colonic carcinoma, is associated with a sense of incomplete evacuation of the bowel (tenesmus).
Investigations for suspected colonic malignancy includes full blood cell count, renal function and electrolytes, liver function tests (to rule out hepatic involvement), plain X-ray of the abdomen, and ultrasound and CT scans. Carcinoembryonic antigen (CEA) is the commonly used tumour marker to diagnose colonic malignancy and subsequently to assess the progress, including recurrence and treatment.

Case Scenario 3

Background : A 49y old gentleman presents to A&E with an 8h history of severe epigastric and central abdominal pain radiating to his back. The pain reduces when he leans forward and he also reports three episodes of bilious vomiting.

Social History :He lives alone and is a smoker and drinks up to 50-60 units per week. He is vague about his appetite and support at home. He also states experiencing similar episodes like this before but with less severe pain.

Examinations : Temperature = 37.6°C, heart rate is 92bpm and respiratory rate of 18bpm. Abdominal examination reveals tenderness over the epigastric region with some guarding but no evidence of peritoneal irritation.

Investigations :Plain radiographs of the chest (erect) and abdomen (supine) are unremarkable.
Bloods :U&Es: normal, Haemoglobin = 140g/L (N), WCC: 16.8 (H), C-reactive protein 215mg/L (high), Corrected Calcium: 2.05mmol/L (L)
Imaging : Plain abdominal X-ray reveals a dilated descending and transverse colon

Case Scenario 3

On review of the patient what is the most likely diagnosis?



Consider a provisional diagnosis and work your way through the content to help revise the above clinical conditions before confirming your answer.

Intestinal Obstruction Definition: Intestinal obstruction can broadly be categorised by the location of the obstruction, either within the small or large intestine and via the cause e.g intra/extra luminal or intrinsic.
This is in reference to the site of the bowel wall of which is causing the decreased passage of matter. Always consider presenting symptoms as they can help to distinguish the potential location i.e. whether this is a high obstruction e.g. small bowel obstruction (SBO) or low e.g. large bowel obstruction (LBO). A good understanding of GI anatomy will help with this.

Risk factors:
Intestinal Obstruction
Cardinal Features::
  1. Abdominal distension
  2. Vomiting
  3. Constipation/Obstipation
  4. Colicky pain
Investigations: Plain Abdo XR *note key features
Blood – FBCs, U&Es, CRP, Group and Save
Consider ultrasound scan for SBO
Consider further non-contrast CT
Check the commonest features to the appropriate diagnosis:
Small Bowel Obstruction Large Bowel Obstruction

Intestinal Obstruction Commonly a plain abdominal X-Ray will be requested as a primary investigation for suspected intestinal obstruction. There is classical features that can be visualised and interpreted to help make a diagnosis of the location of the obstruction – SBO vs LBO.

The two abdominal X-Ray films below show SBO and LBO - select the most appropriate features by clicking on your correct answer in the middle table.
Mesenteric Ischaemia
Definition:This is an umbrella term covering a number of conditions, including acute mesenteric arterial embolus and thrombus, mesenteric venous thrombus and non-occlusive mesenteric ischaemia (NOMI). These result in a form of an infarction to the intestines disrupting the blood supply and significantly reducing blood flow thus resulting in ischaemia. The splenic flexure should be noted as a vulnerable region as this is the border of the inferior and superior mesenteric arteries.
Predisposing factors:
  1. Increasing age
  2. Atrial fibrillation
  3. CVD risk factors: smoking, hypertension, diabetes and causes of atherosclerosis (↑lipids)
  4. Other causes of emboli: endocarditis
  5. Drug use e.g. cocaine: highly suspicious in younger patients following cocaine use
Features:
  1. Abdominal pain – out of proportion to physical findings
  2. Rectal bleeding
  3. Diarrhoea
  4. Febrile/Pyrexic
  5. Potentially ↑ white cell count associated with acidosis
Mesenteric Ischaemia
Clinical Features: Presenting features and history may vary widely. Mesenteric ischaemia can also be known as ischaemic bowel disease and also can be separated into 3 more distinct conditions which aim to factor in the time course and anatomical location:
Acute Mesenteric Ischaemia
Chronic Mesenteric Ischaemia
Colonic Ischaemia
Each condition is likely to present in a particular way and history taking needs to be thorough in order confidently make an accurate diagnosis. The history should explore the key characteristics of the abdominal pain, using the mnemonic SOCRATES:
Site
Onset
Character
Radiation
Associations - nausea, vomiting, diarrhoea
Timing, duration, frequency
Exacerbating and relieving factors
Severity
Mesenteric Ischaemia Clinical Features: As presenting features and history vary widely. It is useful to separate out the 3 main diagnosis and be familiar with how they individually present. The table below helps to establish these similarities and differences utilising SOCRATES to help establish an accurate diagnosis.

Mesenteric Ischaemia Once obtaining a history having a good understanding of the blood supply to this region is important. Arterial embolus and infarction can occur across any part of the mesenteric arteries/branches. It is therefore important to be familiar with the relevant branches and distributions divided as Superior Mesenteric Arteries (SMA) and Inferior Mesenteric Arteries (IMA) and to what anatomical regions they supply.

Within the list below drag and drop the correct label to the anatomical illustration. Note the transverse colon has been pulled superiorly to aid understanding.
Mesenteric Ischaemia Arterial embolus and infarction can occur across any part of the mesenteric arteries/branches. It is therefore important to be familiar with the relevant branches and distributions divided as Superior Mesenteric Arteries (SMA) and Inferior Mesenteric Arteries (IMA) and to what anatomical regions they supply.

Within the list below drag and drop the correct label to the anatomical illustration. Note the transverse colon has been pulled superiorly to aid understanding.
Acute Pancreatitis Definition:

Disease of increasing incidence and unpredictability. Although most case are mild (80%) up to 1/5 will develop severe acute pancreatitis with a degree of multiorgan failure. Progression can be rapid and potentially fatal. The pathogenesis is not fully understood but a common pathway is believed to result in elevated intracellular calcium levels which in turn activates various proteases and pancreatic enzymes. This causes inflammation and perpetuates ‘autodigestion’ of the pancreas. The result is pancreatic dysfunction which can be long term if necrosis has occurred.



Acute Pancreatitis                    
Investigations: After the clinical examination this aims to confirm the suspected diagnosis and to assess the severity of the disease and determine the underlying cause.
Bloods:

Raised Amylase - >1000u/mL or x3 the upper limit of normal (may be normal if patient presents late as this declines after 48h (note this can be elevated in other GI pathology e.g. perforation, infarction)
Raised Serum Lipase - Elevated levels more sensitive and specific for diagnosing pancreatitis
U&Es, LFTs, FBC, WCC, CRP, Glucose, Corrected Calcium and ABGs assessed at admission and 24h + 48h to help assess severity and distinguish potential causation.

Radiology:
Erect CXR – to exclude other pathology e.g. perforation, peptic ulceration
CT – contrast enhanced to help make diagnosis and extent of damage (MRI maybe preferable in complex cases)
US – warranted if suspecting gallstones (often seen with elevated LFTs - ↑AST specifically)

Assessing for Severity - This helps to determine treatment and optimal setting to provide care. This can be done via various methods but commonly used is the Modified Glasgow Criteria
Acute Pancreatitis                    
Modified Glasgow Criteria for Assessing Severity
If there are ≥3 criteria met within the initial 48h then should initiate prompt management in ITU/HDU setting A useful mnemonic to remember this is PANCREAS

P aO2 < 8kPa
A ge >55y
N eutrophillia – WCC >15 x109/L
C alcium - < 2mmol/L
R enal function – Urea >16mmol/L
E nzymes – LDH>600iu/L; AST>200iu/L
A lbumin < 32g/L
S ugar – assess blood glucose >10mmol/l
Perforated Peptic Ulcer                            
Definition: Peptic ulcer is a term which covers both gastric and duodenal ulcers. Ulcerations along the GI tract have various causes but ultimately form due to an alteration to the normal defence mechanisms which include mucus, bicarbonate, mucosal blood flow and prostaglandins. The presence of Helicobacter pylori (Gram negative bacteria) impacts the level of acid in the stomach and is associated with peptic ulcer disease (95% of duodenal ulcers, 75% of gastric ulcers) and is therefore important to investigate. Perforation can occur when these are not diagnosed or managed effectively. This occurs due to the ulcer ‘burning’ through the gastric or duodenal mucosa and allowing digestive juices and enzymes into the abdominal cavity – this usually requires immediate surgery.
Perforated Peptic Ulcer                            
Specific Signs of Perforation:
Most characteristic symptom – sudden onset of epigastric pain, nausea, dyspepsia
Generalised across the abdomen – may radiate to RLQ
Upper digestive bleeding can be present due to erosion into nearby vasculature e.g. gastroduodenal artery
Signs of peritonism may occur is the ulcer fails to seal – board like rigidity, hypotension and fever can be signs of shock and bacterial peritonitis


Duodenal ulcers are associated with increased age and are 4 time more common the gastric ulcers?

The Drug class prevent peptic ulcer disease works on lesser gastric acid secretion via binding to the proton pump of the gastric parietal cell

Ruptured Aortic Aneurysm                            
Definition:
Abdominal Aortic Aneursym (AAA) are the most common form of rupture and this risk increases significantly with age. An aneurysm causes abnormal dilatation of the vessel beyond 50% of its usual size. This can be subdivide into
True aneurysm: All layers of the arterial wall are dilated
False aneurysm: Whereby a collection of blood is trapped in the outer adventitia causing dilation.
Predisposing factors:
Gender – males 6 fold more common vs females
Age – 95% occur in >65y
Known atheroma
Trauma
Infections
Connective tissue disorders (e.g. Marfan’s, Ehtlers-Danlos)
Inflammatory conditions and vasculitis
Features:
Rupture is more likely to occur when >6cm across
Intermittent or continuous abdo pain
Pain radiates to back/iliac fossa
Expansile abdominal mass
Collapse
Clinical shock
UK screening Programme
Eligible for men ≥65y for US monitoring of AAA Click here fore more info
Think back to case scenario 3. Accounting for the highlighted features within the history and investigations a probable diagnosis is?
Background:A 49y old gentleman presents to A&E with an 8h history of severe epigastric and central abdominal pain radiating to his back. The pain reduces when he leans forward and he also reports three episodes of bilious vomiting.

Social History: He lives alone and is a smoker and drinks up to 50-60 units per week. He is vague about his appetite and support at home. He also states experiencing similar episodes like this before but with less severe pain.

Examinations: Temperature = 37.6°C, heart rate is 92bpm and respiratory rate of 18bpm. Abdominal examination reveals tenderness over the epigastric region with some guarding but no evidence of peritoneal irritation.

Investigations: Plain radiographs of the chest (erect) and abdomen (supine) are unremarkable.

Bloods: U&Es: normal, Haemoglobin = 140g/L (N), WCC: 16.8 (H), C-reactive protein 215mg/L (high), Corrected Calcium: 2.05mmol/L (L)

Imaging: : Plain abdominal X-ray reveals a dilated descending and transverse colon.
Case Scenario 3
On review of the patient what is the most likely diagnosis?

Case scenario 3: Explanation
EXPLANATION:

The signs and symptoms in this patient are very suggestive of acute pancreatitis. Alcohol accounts for about 30-35% of all cases of acute pancreatitis.

The other important cause of acute pancreatitis is obstruction secondary to gallstones (30-40%). Approximately 25% of patients presenting with acute pancreatitis may have associated cardiovascular (tachycardia) or respiratory (tachypnoea) symptoms.

Pancreatitis is thought to result from early activation of pancreatic enzymes, producing auto-digestion of the pancreas and surrounding tissues. Various prognostic scores are used to assess the severity. Currently in the UK, the Glasgow-Imrie scoring system is widely used and can help predict the prognosis in acute pancreatitis – this includes the following:

Age>55 years, WCC>15 × 109/l, glucose>10 mmol/l, urea>16 mmol/l, PaO2<60 mm Hg, calcium<2 mmol/l, albumin<32 g/l, lactate dehydrogenase>600 units/l, asparate/alanine aminotransferase>100 units/l.

C-reactive protein although not part of the Glasgow criteria, has an independent prognostic value if the peak level is >210 mg/l in the first four days of the attack

Serum amylase is a useful indicator to diagnose acute pancreatitis; a diagnosis of acute pancreatitis is likely if the level is three times the upper limit of normal.

Ultrasound of the abdomen is indicated in all patients with acute pancreatitis to determine the presence/absence of biliary calculi. A CT of the abdomen should be performed on all patients with severe acute pancreatitis, preferably between days 3 - 10 following the onset of symptoms, to rule out pancreatic necrosis.
Case Scenario 4

Background : A 42-year-old barmaid presents to the Accident and Emergency department with an 8-hour history of severe right upper quadrant pain and vomiting. She says that the pain is radiating to her right scapula and is exacerbated on breathing. She appears pale and mildly jaundiced.

Social History :Barmaid, Alcohol intake approx. 10 units/wk, smoker with a 20y pack history, BMI: 32.2kg/m2 (obese category)

Examinations : : Pulse rate is 96/min, blood pressure is 126/82 mmHg and temperature is 37.7º C. Abdominal examination reveals tenderness over the right hypochondrium but no mass is palpable. When palpating her RUQ she winces with pain reporting right shoulder pain.

Bloods :Na: 142mmol/l (N), K: 3.3mmol/L, Bil: 45µmol/L (H), ALT: 54iU/L (H), AST: 65iU/L (H), WCC: 18 x 109/L (H)
Imaging : Plain radiographs of the abdomen (supine) and chest (erect) are unremarkable.

Case Scenario 4
On review of the patient what is the most likely diagnosis?



Consider a provisional diagnosis and work your way through the content to help revise the above clinical conditions before confirming your answer.

Ascending Biliary Cholangitis Commonly a plain abdominal X-Ray will be requested as a primary investigation for suspected intestinal obstruction. There is classical features that can be visualised and interpreted to help make a diagnosis of the location of the obstruction – SBO vs LBO.

Definition: This is a systemic bacterial infection effecting the biliary tree resulting in localised inflammation and infection. The most common cause is due to obstruction within the common bile duct (CBD) by the presence of gallstones (choledocholithiasis).

Causes: The obstruction causes cholangiovenous and cholangiolymphatic reflux due to increased intra-biliary pressures. Other causes can be due to the bacterial translocation from the duodenum up the biliary tree and into the bloodstream.

Revision of the Biliary Tree – drag and drop the below to the corresponding anatomical position:
Ascending Biliary Cholangitis Commonly a plain abdominal X-Ray will be requested as a primary investigation for suspected intestinal obstruction. There is classical features that can be visualised and interpreted to help make a diagnosis of the location of the obstruction – SBO vs LBO.

Clinical features:
Fever is the most common feature occurring in 90% of patients
Charcot's triad of right upper quadrant (RUQ) pain, fever and jaundice occurs in about 20-50% of patients
RUQ pain 70%
Jaundice 60%
Hypotension and confusion are also common and indicate severe disease

Investigations:
WCC:To determine extent of sepsis (usually elevated)
Blood cultures: Common causative organisms e.g. E.coli, Enterococcus
LFTs: Cholestatic picture → ↑ALP and ↑Bilirubin
US: CBD will often be dilated with thickened wall
MRCP: Assess the site and cause of obstruction
ERCP: Definitive investigation and can aid drainage and enables sampling of obstruction
Acute Cholecystitis
Definition: Condition which follows the impaction of a stone or ‘sludge’ at the neck of the gallbladder or along the cystic duct. Very rarely can occur without the presence of a stone called acalculous cholecystitis. Partial obstruction, short in duration results in biliary colic BUT when prolonged complete obstruction over many hours this leads to acute cholecystitis

Clinical features:
 Continuous epigastric or RUQ pain – referred to the R shoulder
 Vomiting
 Fever – febrile (with ↑WCC vs biliary colic whereby WCC are normal
 Local peritonism – abdominal examination = guarding and the below signs

Acute Cholecystitis
Investigations:
WCC: Elevated in the case of infection or inflammation
Plain AXR: Can indicate approx. 10% of gallstones
Abdominal US: Can visualise the gallstones and see a distended gallbladder or sometimes shrunken in chronic disease. Thickened wall (>3mm) is often observed along with dilated CBD (>6mm)
Acute Pancreatitis – Revision Answers Definition: Disease of increasing incidence and unpredictability. Although most case are mild (80%) up to 1/5 will develop severe acute pancreatitis with a degree of multiorgan failure. Progression can be rapid and potentially fatal. The pathogenesis is not fully understood but a common pathway is believed to result in elevated intracellular calcium levels which in turn activates various proteases and pancreatic enzymes. This causes inflammation and perpetuates ‘autodigestion’ of the pancreas. The result is pancreatic dysfunction which can be long term if necrosis has occurred.

Perforated Peptic Ulcer – Revision fill in the gaps

Specific Signs of Perforation:

  • Most characteristic symptom – sudden onset of epigastric pain, nausea, dyspepsia
  • Generalised across the abdomen – may radiate to
  • Upper digestive bleeding can be present due to erosion into nearby vasculature e.g. gastroduodenal artery
  • Signs of peritonism may occur is the ulcer fails to seal – board like rigidity, hypotension and fever can be signs of shock and bacterial peritonitis

Right Lower Lobe Pneumonia Definition: Acute lower tract infection usually of bacterial origin. This causes inflammation of the substances of the lungs and can be classified anatomically e.g. lobar (whereby an entire pulmonary lobe is effected) or by its aetiology. There are many causes to pneumonia and can be broadly categorised as below:

 Community - acquired
 Hospital – acquired
 Aspiration
 Immunocompromised

Clinical Features: These will vary dependant on the infecting organism and the baseline health of the patient. General observations and clinical circumstances may help to guide your diagnosis. Generalised symptoms can include:

 Cough +/- purulent sputum
 Fever, rigors
 Pleuritic pain
 Cyanosis
Respiratory examination
Signs specific for pneumonia can include:
 Reduced chest expansion
 Dull percussion
 Bronchial breathing
 ↑ vocal resonance
Right Lower Lobe Pneumonia Investigations:
CXR: Assess for infiltrates, lobar or multilobar and extent of consolidation
Sputum sample: Identify organism via culture and address with appropriate antibiotic
Blood tests: FBC, WCC and CRP

Severity Score – CURB-65
A validated scoring system to help determine appropriate management and admission criteria.
Confusion (AMT ≥8)
Urea (>7mmol/L)
Respiratory rate (≥30bpm)
Blood pressure (< 90mmHg SBP or < 60mmHg DBP)
Each criteria present scores 1 (and +1 for age ≥65y) Treatment pathway
Score 0-1: Consider treatment at home
Score 2: Admit for hospital treatment
Score ≥3: Severe pneumonia likely ITU admission Blood tests: FBC, WCC and CRP
Think back to case scenario 4. Accounting for the highlighted features within the history and investigations a probable diagnosis is?
Background:: A 42-year-old barmaid presents to the Accident and Emergency department with an 8-hour history of severe right upper quadrant pain and vomiting. She says that the pain is radiating to her right scapula and is exacerbated on breathing. She appears pale and mildly jaundiced.

Social History: Barmaid, Alcohol intake approx. 10 units/wk, smoker with a 20y pack history

Examinations: Pulse rate is 96/min, blood pressure is 126/82 mmHg and temperature is 37.7º C. Abdominal examination reveals tenderness over the right hypochondrium but no mass is palpable. When palpating her RUQ she winces with pain reporting right shoulder pain.

Bloods: Na: 142mmol/l (N), K: 3.3mmol/L, Bil: 45µmol/L (H), ALT: 54iU/L (H), AST: 65iU/L (H), WCC: 18 x 109/L (H)

Imaging: Plain radiographs of the abdomen (supine) and chest (erect) are unremarkable.
Case Scenario 4
On review of the patient what is the most likely diagnosis?

Case scenario 4: Explanation
EXPLANATION:

The history, signs and symptoms in this patient are suggestive of acute cholecystitis. Acute cholecystitis is more common in females over the age of 40 and with high BMI.

Gallstones are the commonest cause for acute cholecystitis. Obstruction of the common bile duct due to stones leads to accumulation of bile and inflammation, resulting in an acutely inflamed gall bladder.

Other risk factors for acute cholecystitis include alcohol abuse and tumours of the gall bladder. The signs and symptoms of acute cholecystitis include severe right hypochondrial pain exacerbated by respiration, nausea and vomiting, and increase in temperature. The rise in temperature is frequently mild to moderate; a very high temperature with or without chills and rigors may point to a diagnosis of acute cholangitis – usually caused by a bacterial infection.

A tender, inflamed gall bladder may be palpable in some patients and can be tested to assess if the patient is Murphy’s sign positive. Likewise, jaundice may or may not be present whereby in this case mild jaundice was observed with a mildly elevated bilirubin level. Abdominal and chest X-Rays were taken but in practice abdominal ultrasound was also clinically indicated.

The differential diagnoses for acute cholecystitis include acute pancreatitis, peptic ulcer disease or perforated peptic ulcer, appendicitis, acute infective hepatitis and pleurisy.
Case Scenario 5

Background :A 52-year-old estate agent presents to his General Practitioner with a four-week history of upper abdominal pain and diarrhoea. He says that he has also had a few episodes of ‘dark-coloured’ vomit during this period. General examination is unremarkable.

Social History :Estate agent, non-smoker, alcohol intake <15 units/wk

Examinations : Abdominal examination indicates severe epigastric pain but is otherwise unremarkable

Bloods :Hb: 115g/L (L), MCV: 68fL (L), WCC: 6 x109/L (N), CRP: <5mg/L (N)

Investigations :Endoscopic examination of his upper gastro-intestinal region reveals multiple ulcers in the stomach and duodenum.

Case Scenario 5
On review of the patient what is the most likely diagnosis?



Consider a provisional diagnosis and work your way through the content to help revise the above clinical conditions before confirming your answer.

Vasoactive intestinal peptide tumour
Definition: Also known as VIPomas are rare functioning neuroendocrine tumours that secrete vasoactive intestinal polypeptide (VIP). They are detected in 1 in 10 million people per year and most commonly arise in the pancreas as a  functioning pancreatic neuroendocrine (islet cell) tumour accounting for 95%. VIPomas are usually present at 2 main age groups and thus diagnosed in adulthood between 30 - 50 years of age but equally between 2 – 4 years of age. The tumours commonly present as isolated masses secreting excessive amounts of VIP.

Action of VIP: Excessive VIP causes fluid and electrolyte secretion into the gut lumen, resulting in secretory diarrhoea and losses of potassium causing hypokalemia. Other notable clinical actions include; vasodilation, inhibition of gastric acid secretion, bone resorption and enhanced glycogenolysis responsible for flushing and hyperglycaemia.

Clinical Features :


Secretory diarrhoea 700ml – 3000ml per day (odourless and ‘tea’ coloured)
Flushing
Dehydration
Nausea and vomiting
Hyperglycaemia
Symptoms relating to hypokalaemia e.g. muscle weakness/cramping
Faltering growth in children
Vasoactive intestinal peptide tumour
Investigations and Diagnosis:
U&Es: Assessing for electrolyte abnormalities and hydration status
Stool osmotic gap: Determines the cause of diarrhoea (account for stool Na+ and K+ levels)
Vasoactive intestinal polypeptide (VIP) concentration: >75 pg/mL is diagnostic but levels can vary between 2-10 times the normal value
Imaging: This is normally conducted when there is high clinical suspicion. CT or MRI of the abdomen can localize the tumour and stage the extent of disease which can help determine treatment
Tumour of the tail of the pancreas as seen on CT scan of the abdomen. A needle is seen being passed into the tumour for biopsy.
ACTH secreting tumour
Definition: This often presents as bilateral adrenal hyperplasia caused by a ACTH secreting tumour in the pituitary, usually in the form of a benign microadenoma. Up to 70 percent of people with Cushing's syndrome have these benign pituitary tumours whereby they produce excess amounts of ACTH, the hormone that stimulates the adrenal gland to produce cortisol. This condition is called Cushing's disease, which shouldn't be confused with Cushing's syndrome. Most of these tumours are very small, and they may be difficult to identify.
Incidence: Rare (incidence ~ 2million) BUT high mortality if untreated Most common age 20-40y and more common in females 15:1

Pathophysiology: The pituitary adenomas (micro/macro) cause increased secretions of ACTH. This acts within the hypothalamic, pituitary, adrenal (HPA) axis causing excessive production cortisol from the adrenal gland. It is the increased cortisol which causes ‘cushingoid’ features and the clinical presentation. It is therefore important to recall the role and physiological consequence of this hormone.
ACTH secreting tumour
Cortisol: TLipid soluble hormone known as a glucocorticoid. Its is produced in the zona fasciculata of the adrenal cortex. This crosses the cellular membranes and binds to intracellular receptors across the body to exert its actions. Illness resulting from excessive cortisol secretion is called hypercortisolaemia and is apparent in Cushing’s disease.
Actions of cortisol:


Increase in hepatic gluconeogenesis → elevated blood glucose
Inhibition of T and B cell immune responses → reduced immunity
Inhibition of the synthesis and secretion of cytokines
Redistribution of body fat → central obesity and ‘buffalo hump’
Reduced collagen synthesis in the skin → causing purple straie ‘stretch marks’
Decrease in protein synthesis → decreased muscle mass/ulcer formation
Increased osteoclast activity in bone → ↓ BMD and ↑ fracture risk
Increase in BP via increased vascular sensitivity to catecholamines
ACTH secreting tumour
Investigations:
Bloods: FBCs, U&Es, LFTs, Glucose, lipid profile and diagnostic testing with cortisol
Radiology: CXR – lung and vertebral collapse
Other Imaging: MRI pituitary – 70% abnormal in Cushing’s disease. A CT chest/abdomen would be prudent to rule out any ectopic tumours
Diagnosis: Some of the biochemical diagnostic tests used include salivary and blood serum cortisol testing. 24-hour urinary free cortisol (UFC) testing, the dexamethasone suppression test (DST). No single test is perfect and multiple tests will be required for an accurate diagnosis. The Dexamethasone suppression test - low dose and high dose can be used to observe is cortisol production can be supressed. Cortisol levels are measured at 0h and 48h post administration of dexamethasone. If this remains elevated this is indicates the presence of Cushing’s. In Cushing’s disease ACTH will also be elevated as exogenous steroids fail to suppress the action of the tumour. Diagnosing Cushing's disease is a multidisciplinary process involving doctors, endocrinologists, radiologists, surgeons, and chemical pathologists
Somatostatinoma
Definition: Somatostatinoma is a rare malignant neuroendocrine tumour of the delta cells of the pancreas that produces somatostatin in excessive amounts. This most commonly diagnosed in the age group of 50-55y with an equal male:female distribution. 45 percent of somatostatinomas occur in association with multiple endocrine neoplasia MEN-1 syndrome (remember as the 3Ps). Symptoms relate to the excessive somatostatin.
Clinical Features:
Most common - - abdominal pain and weight loss are present.
Less common - - patients present with somatostatinoma syndrome:
Diabetes mellitus or glucose intolerance
Cholelithiasis
Diarrhoea/steatorrhoea
Carcinoid tumour
Definition: The term "carcinoid" is generally applied to well-differentiated neuroendocrine tumours originating in the GI tract, lungs, or rare primary sites, such as the kidneys or ovaries. Carcinoid tumours are apudomas that arise from the enterochromaffin cells throughout the gut. Over two-thirds of carcinoid tumours are found in the GI tract most commonly the ileum, rectum and also the appendix. Features of the carcinoid syndrome may be present in patients with tumours that have metastasized, especially to the liver. The metastatic potential correlates to the size, location and histology.
Clinical Features::
 Gastric tumours are sub classified into 3 types that have different clinical presentation and malignant potential. Commonly anaemia and abdominal pain are present and the role of gastrin should be considered
 Small intestinal carcinoids often present with abdominal pain and/or intermittent obstruction.
 Appendiceal carcinoids are the most common neoplasm found in the appendix; they are most often discovered incidentally during appendectomy.
 Transverse/descending colon and rectal carcinoids may present with changes in bowel habit, obstruction, or fresh bleeding.

Carcinoid syndrome:
Includes flushing and diarrhoea and less frequently heart failure and bronchoconstriction.
Caused by endogenous secretion of mainly  serotonin  and kallikrein.
Gastrinoma
Definition: Gastrinoma is often also referred to as Zollinger-Ellison (ZES) syndrome. This is characterized by gastric acid hypersecretion resulting in severe acid-related peptic disease and diarrhoea. The gastrin secreting tumours usually occur in the duodenum or pancreas. Around 30% occur as part of MEN type I syndrome. Most patients are diagnosed between the ages of 20 – 50y with a higher incidence in men as compared with women.
Clinical Features:

Approximately 10% of patients, especially with metastatic disease or MEN type 1 have symptoms due to a second hormonal syndrome (e.g. VIPoma, somatostatinoma, ACTH secreting tumours)
Diagnosis
Clinical examination, endoscopy confirming refractory peptic ulcers
Gastrin levels should be measured – either basal or stimulated (value >10 times the upper limit of normal (1000 pg/mL) in the presence of gastric acid (gastric pH <2) is diagnostic.
CT and MRI scans post endoscopy are required to confirm tumour location and staging
Case Scenario 5
On review of the patient what is the most likely diagnosis?

Think back to case scenario 5. Accounting for the highlighted features within the history and investigations a probable diagnosis is?
Background:A 52-year-old estate agent presents to his General Practitioner with a four-week history of upper abdominal pain and diarrhoea. He says that he has also had a few episodes of ‘dark-coloured’ vomit during this period. General examination is unremarkable.
Social History: Estate agent, non-smoker, alcohol intake <15 units/wk
Examinations: Abdominal examination indicates severe epigastric pain but is otherwise unremarkable
Bloods: Hb: 115g/L (L), MCV: 68fL (L), WCC: 6 x109/L (N), CRP: <5mg/L (N)
Investigations: Endoscopic examination of his upper gastro-intestinal region reveals multiple ulcers in the stomach and duodenum.
Case scenario 5: Explanation
EXPLANATION:

Gastrinomas primarily occur in the pancreas and duodenum, and are malignant in nearly two-thirds of cases. The patients may present with upper abdominal/epigastric pain and vomiting; the vomittus may be ‘coffee-ground’ due to bleeding from the ulcers.

Ninety percent of patients with gastrinomas develop peptic ulceration. Gastrinomas may either arise sporadically or as part of Zollinger-Ellison syndrome (peptic ulceration, gastric acid hypersecretion and islet cell tumour of the pancreas). Sporadic Zollinger-Ellison occurs most frequently in the fifth decade of life. Approximately 20% of patients with Zollinger-Ellison syndrome have MEN type I syndrome.

An elevated basal gastric acid output >15mEq/h and a serum gastrin >1000pg/ml are suggestive of a gastrinoma. If it is difficult to make a diagnosis, a secretin stimulation test may be indicated. Lesions are localised by somatostatin-receptor scintography. A CT scan may be indicated to exclude metastases.

The treatment of this condition is either conservative (high dose proton pump inhibitors) or surgical. Surgical resection class="case-section" may be aided by intra-operative ultrasound and/or intra-operative endoscopy.
Case Scenario 6

Background :A 69-year-old man is brought to the Accident and Emergency department with severe peri-umbilical pain radiating to the back. He is strikingly tall.

Social History :Retired office manager, ex-smoker (note 30 year pack history), alcohol intake <15units per week, reports active lifestyle - plays golf twice a week

Examinations : On examination, he is pale, sweaty, anxious and cyanosed. His blood pressure is 80/70 mmHg and his pulse rate is 140/min. The rhythm of the pulse is regular. No other abnormalities were detected. Bowel sounds are normal.

Bloods :U&Es: normal, Haemoglobin = 100g/L (L), WCC: 8.0 (N), C-reactive protein 50mg/L (high), eGFR: 75 (L),

Investigations :The junior clerking the patient seeks advice from his seniors, they report an urgent ultrasound would be of benefit and potentially further imaging

Case Scenario 6
On review of the patient what is the most likely diagnosis?



Consider a provisional diagnosis and work your way through the content to help revise the above clinical conditions before confirming your answer.

Ureteric Colic
Definition: This term is often used interchangeably with renal colic. This describes an acute pain caused by the obstruction of a segment in the urinary tract – usually a stone arising and obstructing the ureter. Urolithiasis is the term used for stone formation and is usually asymptomatic but can cause pain when they are displaced and lead to a blockage. The consequences are increased pressure and tension in the ureter and the kidney causing abrupt severe and colicky pain.

Causes: Ureteric colic is generally caused by stones in the ureter obstructing the flow of urine. Less common causes include blood clots (that may develop with upper urinary tract bleeding) or sloughed renal papilla (for example due to diabetes).

The obstruction causes increased tension in the urinary tract wall, stimulating the production of prostaglandins leading to vasodilation and smooth muscle to spasm. The increased vasodilation leads to diuresis exacerbating the pressure within the renal system. Spasms present as a colicky pain in the loin or flank and radiating to the labia in women or to the groin or testicle in men.
Ureteric Colic
Revision of basic kidney anatomy and the course of the urinary tract can help to identify common areas in which stones are likely to form and obstruct. Within the list below drag and drop the correct label to the anatomical illustration.

Ureteric Colic
Risk Factors:
Dehydration —reduced urine output <1 litre a day.
Urine pH — This impacts various stone formation low pH ↑ risk of uric acid stones and high pH ↑ risk of calcium phosphate stones
Age and gender — most common between ages of 40-60years
Family history of stone formation — there is a 2.5 times greater risk in people with a family history
Obesity — independent risk factor for stone formation
Diet — excessive oxalate, urate, sodium and animal protein
Medications — commonly protease inhibitors and diuretics e.g. furosemide
Anatomical abnormalities of the urinary tract 
Clinical Features:
Severe unilateral abdominal pain originating in the loin or flank and radiating to groin region
Abrupt colicky pain
Nausea & vomiting +/- haematuria
Described as the most severe pain experienced by the person ‘more intense than that of childbirth’
Pyrexia and sweats — if concomitant urinary infection is present
Dysuria, urinary frequency, and straining — when the stone reaches the vesico-ureteric junction (due to the stone irritating the detrusor muscle).

Diagnosis:
This can done by via the patient history and physical examination. A urine dipstick is always essential to check for underlying infection (nitrates/leukocytes/blood). Exclude common medical risk factors for urinary stones by checking bloods - calcium, phosphate, and urate levels along with renal function
Imaging may be required -non-contrast helical CT to confirm if stones will spontaneously pass
Crohn’s Disease - Revision
Select the correct answer to the short questions below:

Crohn’s disease is often described on endoscopy as having what appearance: Crohn’s disease is often observed to have granulomas present, these are: Where is the most common anatomical locations of the GI tract in which Crohn’s effects: Individuals with Crohn’s disease are at greater risk of bowel obstruction: The inflammation in Crohn’s effects all layers of the intestinal wall and is said to be transmural:
Crohn’s Disease
Definition: Crohn’s disease is a chronic relapsing and remitting inflammatory condition affecting any part of the GI tract from the mouth to the anus. It is characterised by transmural inflammation with non-caseating granulomas and on endoscope a ‘cobblestone’ appearance can be observed. The disease can affect any part of the gastrointestinal tract from the mouth to the anus, although it frequently affects the terminal ileum (terminal ileitis). Inflammation occurs in all layers, down to the serosa increasing the risk of complications such as  strictures, fistulas and adhesions
Risk Factors:
The precise aetiology is unknown but an altered response by the body’s immune system to normal intestinal bacteria are implicated. Factors therefore include:
 Smoking (smokers have a x3-4 ↑ risk vs non-smokers)
 Environmental factors
 Familial predisposition
 Theories linking pathogenic bacteria and viruses has all been implicated
Crohn’s Disease -Revision - fill in the gaps
Presentation:
 Non bloody diarrhoea with ↑ frequency/urgency
 Tenesmus
 Abdominal pain/tenderness – dependant on where inflammation occurs
 Weight loss/malaise/fever
 Perianal skin tags, abscess
 Fistula and strictures may be present
Investigations:
 Bloods: FBC, CRP, ESR, Ferritin
 Stool culturing – checking for common pathogens e.g. E.Coli, C.Difficile, Campylopacter
 Colonoscopy +/- rectal biopsy
 MRI – can assess pelvic disease and locate strictures/fistula
Colonoscopy showing continuous ‘cobblestone’ appearance of the colon
Adhesive Small Bowel Obstruction               Definition: Intra-abdominal adhesions following abdominal surgery represent a major unsolved problem. ASBO after peritoneal cavity surgery is a well-known disease entity and is one of the most common causes of small bowel obstruction. As with intestinal obstruction this disrupts luminal flow due to fibrous banding formation which bind separate parts of the intestine together. These can later contract thus narrowing the lumen – forming an obstruction. These adhesion can result in significant morbidity, mortality and infertility in women, and adhesion-related complications for future surgery.
Risk Factors:
 Prior abdominal surgery sig. ↑ of open surgery vs laparoscopic
 Pelvic surgery and peritoneal damage
 Abdominal wall or groin hernia
 Intestinal inflammation
 History of, or increased risk for neoplasm
 Prior irradiation


Adhesive Small Bowel Obstruction
Clinical Features:
 Severe dehydration
 Bilious Vomit
 Central Colicky Pain
 Electrolyte abnormalities
 Early Vomiting
 Signs of peritonitis (severe)
Treatment: Dependent on the complexity and severity.

Less severe
Partial obstruction may resolve with conservative management e.g. NGT for decompression of gas ‘drip and suck’ i.e. IV fluids and close observations.

Severe
Operative management Laparoscopic/open surgery to remove adhesion
Ruptured Aortic Aneurysm - Revision
Definition: Abdominal Aortic Aneurysm (AAA) are the most common form of rupture and this risk increases significantly with age. An aneurysm causes abnormal dilatation of the vessel beyond 50% of its usual size. This can be subdivide into

True aneurysm: All layers of the arterial wall are dilated
False aneurysm: Whereby a collection of blood is trapped in the outer adventitia causing dilation.
Predisposing factors:
 Gender – males 6 fold more common vs females
 Age – 95% occur in >65y
 Known atheroma
 Trauma
 Infections
 Connective tissue disorders (e.g. Marfan’s, Ehtlers-Danlos)
 Inflammatory conditions and vasculitis
Features:
 Rupture is more likely to occur when >6cm across
 Intermittent or continuous abdominal pain
 Pain radiates to back/iliac fossa
 Expansile abdominal mass
 Collapse
 Clinical shock e.g. hypovolemic/cardiogenic

UK screening Programme
Eligible for men ≥65y for US monitoring of AAA Click here fore more info
Acute Pancreatitis                     Definition: Disease of increasing incidence and unpredictability. Although most case are mild (80%) up to 1/5 will develop severe acute pancreatitis with a degree of multiorgan failure. Progression can be rapid and potentially fatal. The pathogenesis is not fully understood but a common pathway is believed to result in elevated intracellular calcium levels which in turn activates various proteases and pancreatic enzymes. This causes inflammation and perpetuates ‘autodigestion’ of the pancreas. The result is pancreatic dysfunction which can be long term if necrosis has occurred.

Acute Pancreatitis -Revision        
Think back to case scenario 6. Accounting for the highlighted features within the history and investigations a probable diagnosis is?
Background:A 69-year-old man is brought to the Accident and Emergency department with severe peri-umbilical pain radiating to the back. He is strikingly tall.
Social History: Retired office manager, ex-smoker (note 30 year pack history), alcohol intake <15units per week, reports active lifestyle - plays golf twice a week
Examinations: On examination, he is pale, sweaty, anxious and cyanosed. His blood pressure is 80/70 mmHg and his pulse rate is 140/min. The rhythm of the pulse is regular. No other abnormalities were detected. Bowel sounds are normal.
Bloods: U&Es: normal, Haemoglobin = 100g/L (L), WCC: 8.0 (N), C-reactive protein 50mg/L (high), eGFR: 75 (L),
Investigations: The junior clerking the patient seeks advice from his seniors, they report an urgent ultrasound would be of benefit and potentially further imaging
Case Scenario 6
On review of the patient what is the most likely diagnosis?

Case scenario 6: Explanation
EXPLANATION:

Abdominal aortic aneurysm is one of the commonest form of aneurysm in middle-aged and elderly patients. It is more common in males compared to females.

The risk factors include smoking, hypertension a family history, and increasing age and tall and slim built. The patient may be asymptomatic before the rupture and may present with a pulsatile mass. As the aneurysm increases in size the patient may present with lower back ache.

Ruptured AAA presents with periumbilical pain that radiates to the back. Associated symptoms include hypotension, dizziness, lack of orientation and tachycardia.

The common investigations undertaken done to confirm the diagnosis include abdominal ultrasound, CT or a MRI scan.

The treatment plan is based on the size of the aneurysm. For an aneurysm that is less than 5-cms, observation and conservative treatment (e.g., using anti-hypertensive drugs) will suffice. For aneurysms greater than 5-cms in diameter, elective surgery may be indicated (resection class="case-section" of aneurysm and insertion of a synthetic graft).
Case Scenario 7

Background :A 70-year-old lady who is a taking regular treatment for arthritis is brought to the Accident and Emergency department with sudden onset epigastric pain. She is taking ibuprofen for osteoarthritis.

Social History :She live with her husband and is his main carer at home. Her mobility has been reduced for the last 2 months due to her pain secondary to her osteoarthritis. She is a non-smoker and reports minimal alcohol intake.

Examinations : On examination of her abdomen she generalised tenderness reporting sig. pain to the RLQ and is guarded when you attempted deep palpation. Observations show she is apyrexial, her blood pressure is 110/80 mmHg and her pulse rate is 110/min (regular). A per rectal examination reveals malena.

Bloods :U&Es: normal, Haemoglobin = 100g/L (L), WCC: 7.5 (N), C-reactive protein <10mg/L (N), eGFR: 70 (L),

Investigations: Plain abdominal X-ray is normal. An erect chest X-ray shows gas under the diaphragm.

Case Scenario 7
On review of the patient what is the most likely diagnosis?



: Consider a provisional diagnosis and work your way through the content to help revise the above clinical conditions before confirming your answer.

Ureteric Colic
Risk Factors:
 Dehydration —reduced urine output <1 litre a day.
 Urine pH — This impacts various stone formation low pH ↑ risk of uric acid stones and high pH ↑ risk of calcium phosphate stones
 Age and gender — most common between ages of 40-60years
 Family history of stone formation — there is a 2.5 times greater risk in people with a family history
 Obesity — independent risk factor for stone formation
 Diet — excessive oxalate, urate, sodium and animal protein
 Medications — commonly protease inhibitors and diuretics e.g. furosemide
 Anatomical abnormalities of the urinary tract 
Clinical Features:
With regards to the clinical features of ureteric colic – select if the below statements are true or false:
Pain is often presents abruptly, bilateral and radiating to the shoulder tip? Colicky pain is often described by patients? Pyrexia or sweats often indicate concomitant infection e.g. UTI? Dysuria is a common feature as the stones can often irritate the detrusor muscle?

Crohn’s Disease -Revision
Presentation:
 Non bloody diarrhoea with ↑ frequency/urgency
 Tenesmus
 Abdominal pain/tenderness – dependant on where inflammation occurs
 Weight loss/malaise/fever
 Perianal skin tags, abscess
 Fistula and strictures may be present
Investigations:
 Bloods: FBC, CRP, ESR, Ferritin
 Stool culturing – checking for common pathogens e.g. E.Coli, C.Difficile, Campylopacter
 Colonoscopy +/- rectal biopsy
 MRI – can assess pelvic disease and locate strictures/fistula
Colonoscopy showing continuous ‘cobblestone’ appearance of the colon
Adhesive Small Bowel Obstruction - Revision
Clinical Features:
 Severe dehydration
 Bilious Vomit
 Central Colicky Pain
 Electrolyte abnormalities
 Early Vomiting
 Signs of peritonitis (severe)
Treatment: Dependent on the complexity and severity.

Less severe
Partial obstruction may resolve with conservative management e.g. NGT for decompression of gas ‘drip and suck’ i.e. IV fluids and close observations.

Severe
Operative management Laparoscopic/open surgery to remove adhesion
Perforated Peptic Ulcer – Revision              
Within the statements below select whether they are true or false
Helicobacter pylori is one of the main causes of peptic ulcer and is a gram positive bacteria? The defences in the stomach and GI tract are altered in the presence of  Helicobacter pylori, this is due to ↓inhibition of acid secretion via ↓levels of somatostatin? Gastric ulcers are more common than duodenal ulcers? Epigastric pain before meals and relieved by eating is more likely to be associated with a duodenal ulcer? Gastric ulcers are more commonly found at the greater curve of the stomach?

Perforated Peptic Ulcer – Revision            
Definition: Peptic ulcer is a term which covers both gastric and duodenal ulcers. Ulcerations along the GI tract have various causes but ultimately form due to an alteration to the normal defence mechanisms which include mucus, bicarbonate, mucosal blood flow and prostaglandins. The presence of Helicobacter pylori (Gram negative bacteria) impacts the level of acid in the stomach and is associated with peptic ulcer disease (95% of duodenal ulcers, 75% of gastric ulcers) and is therefore important to investigate. Perforation can occur when these are not diagnosed or managed effectively. This occurs due to the ulcer ‘burning’ through the gastric or duodenal mucosa and allowing digestive juices and enzymes into the abdominal cavity – this usually requires immediate surgery.
Acute Pancreatitis – Revision             Definition: Disease of increasing incidence and unpredictability. Although most case are mild (80%) up to 1/5 will develop severe acute pancreatitis with a degree of multiorgan failure. Progression can be rapid and potentially fatal. The pathogenesis is not fully understood but a common pathway is believed to result in elevated intracellular calcium levels which in turn activates various proteases and pancreatic enzymes. This causes inflammation and perpetuates ‘autodigestion’ of the pancreas. The result is pancreatic dysfunction which can be long term if necrosis has occurred.

Think back to case scenario 7. Accounting for the highlighted features within the history and investigations a probable diagnosis is?
Background:A 70-year-old lady who is a taking regular treatment for arthritis is brought to the Accident and Emergency department with sudden onset epigastric pain. She is taking ibuprofen for osteoarthritis.

Social History: She live with her husband and is his main carer at home. Her mobility has been reduced for the last 2 months due to her pain secondary to her osteoarthritis. She is a non-smoker and reports minimal alcohol intake.

Examinations: On examination of her abdomen she generalised tenderness reporting sig. pain to the RLQ and is guarded when you attempted deep palpation. Observations show she is apyrexial, her blood pressure is 110/80 mmHg and her pulse rate is 110/min (regular). A per rectal examination reveals malena.

Bloods: U&Es: normal, Haemoglobin = 100g/L (L), WCC: 7.5 (N), C-reactive protein <10mg/L (N), Serum Amylase: 140IU/L (N)

Investigations: Plain abdominal X-ray is normal. An erect chest X-ray shows gas under the diaphragm.
Case Scenario 7
On review of the patient what is the most likely diagnosis?

Case scenario 7: Explanation
EXPLANATION:

Perforated ulcer is more common in men than women but it is a common complication following regular analgesic treatment for arthritis. These drugs irritate the gastric mucosa and increased gastric secretion thus causing perforation. It is therefore key to ensure proton pump inhibitors are co-prescribed to help prevent complications – this is not mentioned in this history.

The other risk factors include h. pylori infection, smoking, excessive alcohol consumption, and Zollinger-Ellison syndrome. The majority of symptom includes epigastric pain, nausea and vomiting. The pain may radiate to the back or shoulder tip.

The diagnosis for a perforated peptic ulcer may be made on an erect chest x-ray. There may be evidence of free gas under the diaphragm in case of perforation. There may be elevated serum amylase levels which may not be as raised as in acute pancreatitis. Computerised tomography scan may help with accurate diagnosis.

The complications of perforated peptic ulcer include peritonitis and upper GI bleeding as evidenced by the presence of malena.
Case Scenario 8

Background :A 91-year-old man who lives in residential care is brought to the Accident and Emergency department with a 24-hour history of sudden onset abdominal pain and a change in bowel habits.

Social History :Lives in a residential home with carers. Ex smoker – 40 pack year smoking history. Mobiles with a frame and has no local family members.

Examinations : On examination, he appears pale, cold and clammy. His blood pressure is 98/68 mmHg and his pulse is 94/min and irregular. Abdomen is soft and mildly tender over the umbilical region. The bowel sounds are absent. He was noticed to pass a few episodes of dark (bloody) coloured motions during the day.

Bloods :U&Es: normal, Haemoglobin = 100g/L (L), WCC: 15.0 (H), C-reactive protein 95mg/L (H), eGFR: 55 (L), Analysis of arterial blood gas (ABG) reveals a pH of 7.22, bicarbonate of 18 and a base deficit of -10.

Case Scenario 8
On review of the patient what is the most likely diagnosis?



: Consider a provisional diagnosis and work your way through the content to help revise the above clinical conditions before confirming your answer.

Mesenteric Ischaemia – Revision
Definition: This is an umbrella term covering a number of conditions, including acute mesenteric arterial embolus and thrombus, mesenteric venous thrombus and non-occlusive mesenteric ischaemia (NOMI). These result in a form of an infarction to the intestines disrupting the blood supply and significantly reducing blood flow thus resulting in ischaemia. The splenic flexure should be noted as a vulnerable region as this is the border of the inferior and superior mesenteric arteries.
Predisposing factors:
 Increasing age
 Atrial fibrillation
 CVD risk factors: smoking, hypertension, diabetes and causes of atherosclerosis (↑lipids)
 Other causes of emboli: endocarditis
 Drug use e.g. cocaine: highly suspicious in younger patients following cocaine use
Features:
 Abdominal pain – out of proportion to physical findings
 Rectal bleeding
 Diarrhoea
 Febrile/Pyrexic
 Potentially ↑ white cell count associated with acidosis
Mesenteric Ischaemia – Revision
Clinical Features: As presenting features and history vary widely. It is useful to separate out the 3 main diagnosis and be familiar with how they individually present. The table below helps to establish these similarities and differences utilising SOCRATES to help establish an accurate diagnosis.
Sigmoid volvulus - Revision Definition: Sigmoid volvulus  is a cause of large bowel obstruction and occurs when the sigmoid colon twists on the sigmoid mesocolon. Often when twisting this causes a closed loop of bowel filled with faeces and gas and becomes enlarged. This is the cause of 1/3 of large bowel obstructions in adult but is rare in children.
Clinical presentation:
 Constipation or bowels not opening
 Failure to pass flatus
 Abdominal bloating/distension
 Colicky abdominal pain
 Nausea/vomiting
On examination there is often a palpable, non-tender mass. Per rectal examination highlights an empty rectal ampulla. Abnormal observations e.g. increased HR, temperature >37.5°C and hypotension are a sign of shock and indicate perforation.
Intestinal Obstruction - – Revision
Definition: Intestinal obstruction can broadly be categorised by the location of the obstruction, either within the small or large intestine and via the cause e.g intra/extra luminal or intrinsic. This is in reference to the site of the bowel wall of which is causing the decreased passage of matter. Always consider presenting symptoms as they can help to distinguish the potential location i.e. whether this is a high obstruction e.g. small bowel obstruction (SBO) or low e.g. large bowel obstruction (LBO). A good understanding of GI anatomy will help with this.
  1. Abdominal distension
  2. Vomiting
  3. Constipation/Obstipation
  4. Colicky pain
Clinical features commonly found in SBO vs LBO
Toxic Megacolon – Revision Definition: A complication of IBD – more commonly ulcerative colitis or occasionally in the older population caused by clostridium difficile infection. This is an acute presentation and often a medical emergency. Characterised by excessive colonic dilatation transverse colon usually greater than >6cm
This can cause severe complications:
 Perforation of the colon
 Sepsis
 Shock
Clinical presentation:
 Abdominal distension/bloating
 Abdominal pain
 Dehydration

 Fever (temp >37.5°C)
 Tachycardia (Heart rate >120bpm)
 Hypotension
Typical history e.g. 25-year-old man presenting with abdominal pain, passage of blood and mucus per rectum, abdominal bloating, fever, and disorientation – worsening acutely over the last 2 days. Surgical intervention is often required as treatment
Ruptured Abdominal Aortic Aneurysm - Revision
Definition: Abdominal Aortic Aneurysm (AAA) are the most common form of rupture and this risk increases significantly with age. An aneurysm causes abnormal dilatation of the vessel beyond 50% of its usual size. This can be subdivide into

True aneurysm: All layers of the arterial wall are dilated
False aneurysm: Whereby a collection of blood is trapped in the outer adventitia causing dilation.
Predisposing factors:
 Gender – males 6 fold more common vs females
 Age – 95% occur in >65y
 Known atheroma
 Trauma
 Infections
 Connective tissue disorders (e.g. Marfan’s, Ehtlers-Danlos)
 Inflammatory conditions and vasculitis
Features:
 Rupture is more likely to occur when >6cm across
 Intermittent or continuous abdominal pain
 Pain radiates to back/iliac fossa
 Expansile abdominal mass
 Collapse
 Clinical shock

UK screening Programme
Eligible for men ≥65y for US monitoring of AAA Click here fore more info
Think back to case scenario 8. Accounting for the highlighted features within the history and investigations a probable diagnosis is?
Background:A 91-year-old man who lives in residential care is brought to the Accident and Emergency department with a 24-hour history of sudden onset abdominal pain and a change in bowel habits.

Social History: Lives in a residential home with carers. Ex smoker – 40 pack year smoking history. Mobiles with a frame and has no local family members.

Examinations: On examination, he appears pale, cold and clammy. His blood pressure is 98/68 mmHg and his pulse is 94/min and irregular. Abdomen is soft and mildly tender over the umbilical region. The bowel sounds are absent. He was noticed to pass a few episodes of dark (bloody) coloured motions during the day.

Bloods: U&Es: normal, Haemoglobin = 100g/L (L), WCC: 15.0 (H), C-reactive protein 95mg/L (H), eGFR: 55 (L), Analysis of arterial blood gas (ABG) reveals a pH of 7.22, bicarbonate of 18 and a base deficit of -10.
Case Scenario 8
On review of the patient what is the most likely diagnosis?

Case scenario 8: Explanation
EXPLANATION:

The history, signs and symptoms are classical of mesenteric infarction, also known as ischaemic bowel disease. The majority of the patients who develop ischaemic bowel are elderly and a significant number of them will be in atrial fibrillation.

Clinical features of this condition vary with some patients being asymptomatic or manifesting minimal symptoms during the initial period of developing ischemic bowel; other patients may present with persistent, generalized abdominal pain. Vomiting may or may not be present. Some patients may present with shock; the shock may be out of proportion to the clinical symptoms.

The ischaemic bowel may shed the ‘non-viable mucosa’, which mixed with mucus, results in dark-coloured (also known as ‘plum-coloured) stools. The inflammatory markers such as the white cell count and C-reactive protein may be elevated.

Arterial blood gas analysis is a very useful investigation, which may reveal a metabolic acidosis. This condition is a surgical emergency as the patient rapidly become toxic and may die from septic shock unless the infracted bowel (‘dead gut’) is removed .
Case Scenario 9

Background :Catharine, a female aged 22 years presenting with abdominal pain, was admitted to the surgical unit from the emergency department in a London hospital. She has experienced several episodes of nausea, vomiting and reports having episodes of diarrhoea in the past week which she blamed on her stress and anxiety. Catharine has not eaten for 48 hours and states she feels clammy.

Social History :She is living in student accommodation and had been revising for her upcoming medical school exams which are due to take place in a months’ time. She is a non-smoker and drinks socially, normally on a Wednesday evening at the sports social event but is vague about her unit intake. Her method of contraception is usually the combined oral contraception which she hasn’t been taking for the past 3 months.

Examinations : She reports colicky abdominal pain, on examination you localise this to her right iliac fossa with some guarding. Her skin is clammy with dry mucus membranes evident. She appears dehydrated and has a temp of 37.8°C

Bloods :Na: 147mmol/L (H), Ur: 6.5mmol/L, Cr: 160µmol/L (H), K: 5.4mmol/L (H), Haemoglobin: 130g/L, WCC: 12.5 (H), C-reactive protein 60mg/L (H)

Investigations: :Your senior requests you order a pregnancy test and to consider some appropriate imaging

Case Scenario 9
On review of the patient what is the most likely diagnosis?



: Consider a provisional diagnosis and work your way through the content to help revise the above clinical conditions before confirming your answer.

Acute Appendicitis
Definition: Appendicitis is the inflammation of the vermiform appendix. Located at the at the base of the caecum near the ileocaecal valve. It is likened to a diverticulum of the caecum and thus is susceptible to obstruction often by a faecalith (hard faecal masses), calculi or lymphoid hyperplasia and under increased intraluminal pressure can perforate. This will undergo the usual inflammatory process followed by localized ischaemia which can also form abscesses before perforation. This is the most common cause of acute abdomen in young adults and is most common in the second and the third decade with a 1.5:1 male:female ratio.
Clinical features
Classic triad described in the early onset:
 Periumbilical pain migrating to RLQ (McBurney’s point)
 Nausea and vomiting
 Anorexia

Other common symptoms include:
 Irregular bowel movements/diarrhoea
 Indigestion
 Fever – seen as low grade temperature
Acute Appendicitis    
Investigations: Much of the diagnosis of appendicitis will be made from the clinical history and physical examination. This can be a very acute onset of symptoms and acute peritonitis can be observed in the latter stages which is always indicative of perforation and emergency surgical intervention. However, other tests can be helpful to guide the correct diagnosis:
Blood Tests
Inflammatory markers – CRP, ESR (however, these are non-specific)
Elevated WBC – stated as ‘shift to the left’ in the Alvarado score meaning an increased ratio of immature WBCs to mature WBCs

Imaging
Ultrasound is reliable at identifying abnormal appendixes, especially in thin patients. Often first choice in young adults as lacks ionising radiation
CT is most sensitive (94-98%) and specific (98%) and can rule out other causes of acute abdomen
CT scan (Axial plain) – distended (>6mm) and inflamed appendix with abnormal ‘fat stranding’ grey appearance surrounding it – likely diagnosis of appendicitis
Irritable Bowel Syndrome Definition: This is a chronic functional disorder of the GI tract in the absence of organic disease. Defined as recurrent abdominal pain or discomfort at least three days per month in the last three months with two or more of the following: improvement with defecation, onset associated with a change in frequency of stool, or onset associated with a change in form (appearance) of stool. A positive diagnosis should be sought and rapid exclusion of IBD or coeliac disease is important to minimise further psychological implications to the patient (a well documented trigger). It is thought that up to 15% of adults have symptoms consistent with IBS largely managed in the primary care setting.
Features and subtypes:
IBS with constipation (IBS-C): Patient reports that abnormal bowel movements are usually constipation (type 1 and 2)
IBS with diarrhoea (IBS-D): Patient reports that abnormal bowel movements are usually diarrhoea (type 6 and 7)
Mixed IBS (IBS-M): Patient reports that abnormal bowel movements are usually both constipation and diarrhoea (more than one-fourth of all the abnormal bowel movements were constipation and more than one-fourth were diarrhoea)
Irritable Bowel Syndrome    
Diagnosis:
A diagnosis of IBS should be considered only if the person has abdominal pain or discomfort that is either relieved by defaecation or associated with altered bowel frequency or stool form. This should be accompanied by at least two of the following four symptoms: (NICE 2017)
 Altered stool passage (straining, urgency, incomplete evacuation)
 Abdominal bloating (more common in women), distension, cramps generalised across the abdomen
 Symptoms made worse by eating
 Passage of mucus *but NO blood present*
Other features such as lethargy, nausea, backache and bladder symptoms are common and may be used to support the diagnosis
Investigations:
Full blood count (FBC) – detect anaemia
ESR and/or CRP - to help rule out IBD
Antibody testing – for coeliac disease - endomysial antibodies or tissue transglutaminase
Management
This involves dietary manipulation of fibre, reducing caffeine/fat/alcohol and gas producing foods. Elimination diets e.g. Low FODMAPs can be trialled with expert monitoring by a dietitian
Consideration for managing stress e.g. SSRIs and cognitive behavioural therapy have proven to improve symptoms
Ectopic Pregnancy Definition: When the embryo implants outside of the uterine cavity known as a extrauterine pregnancy which cannot progress and carry a mortality rate of 16.9/100,000 (UK). The most common site of embryo implantation is along the fallopian tube (95%) often the ampulla accounting for 70% of cases. It can also occur at the sites listed below:
 Cornu (2-3%)
 Abdominal cavity (1%)
 Ovary (3%)
 Cervix and caesarean scar (<1%)
Risk Factors:
Often no cause is found but consider any pathology or past medical history that could damage the fallopian tube:  PID usually from an STI
 Tubal surgery
 Previous ectopic
 C-section class="case-section"
 Smokers
 Increased maternal age >35y
Clinical Features:
History:

 Lower abdominal pain in first trimester
 Scanty, dark vaginal bleeding described
 Amenorrhoea 4-10 weeks with pain/abnormal bleed
 Syncopal episodes and shoulder tip pain indicative of intraperitoneal blood loss and rupture
Examination:

 Tachycardia – sign of blood loss
 Abdominal and rebound tenderness
 Pelvic examination showing cervical excitation *avoid adnexal palpation if high suspicious of ectopic as ↑ risk of rupture
Ectopic Pregnancy
Investigations
Human chorionic gonadotropin  Measurement of hCG is performed initially to diagnose pregnancy- this should be performed on all women of reproductive age presenting with abdominal pain or bleeding. The initial test to diagnose pregnancy may be either a urine or serum hCG. Once a pregnancy is confirmed, if ectopic pregnancy is suspected, the serum quantitative hCG is then repeated serially (typically every two days) to assess whether the increase in concentration is consistent with an abnormal pregnancy. Declining or plateauing levels of hCG (<1500mIU/mL) suggest an ectopic or non viable intrauterine pregnancy.
Transvaginal Ultrasound (TVUS) Should be able to detect an intrauterine pregnancy but wont always detect an ectopic. May be able to visualise blood clot ‘free fluid’ in the adnexae or gestation sac with nil fetus within it.
Laparoscopy Is the most sensitive investigation but is often not warranted due to the reliable sensitivity of combining the above investigations. The image on the right shows a laparoscopic view - superiorly to inferiorly in the peritoneal cavity which has been pumped up with carbon dioxide gas to visualize the uterus (marked by blue arrows). On the left Fallopian tube there is an ectopic pregnancy and hematosalpinx (marked by red arrows). The right tube is normal.
Mesenteric Lymphadenitis
Definition: Mesenteric lymphadenitis refers to the inflammation and enlargement of the mesenteric lymph nodes. This is diagnosed in the presence of three or more lymph nodes measuring ≥5mm each and is often initially palpated at the right lower quadrant of the mesentery. This can be caused by various causative organisms which enter the lymphatic system causing hyperplasia, necrosis and pus cells to form which consequently leads to oedematous neighbouring mesentery and precipitates the clinical features.
Causes: Multiple organisms can causes lymphadenitis and will also impact the course of the illness whereby this can be categorised as acute or chronic.
Bacterial causes: Beta-heamolytic streptococcus, Streptococcus viridans, Staphylococcus species and Escherichia coli are the most common causes
Viral causes: Coxsackieviruses (A and B), rubeola virus, and adenovirus have been implicated whilst infectious Epstein-Barr virus (EBV) and human immunodeficiency virus (HIV) have also been observed.
Mesenteric and Adjacent Lymphatics
Mesenteric Lymphadenitis
Clinical Features: Severity of symptoms will often depend on the causative organism and the premorbid status of the individual whereby this often appears most severe at the two extremes of age. In the young this is commonly confused for acute appendicitis where as in the older adult this takes on a more insidious time course.
Signs and symptoms include:

 Abdominal pain – often central or RLQ but may be more diffuse
 Fever – low grade
 Diarrhoea
 Malaise and Anorexia
 Concomitant upper respiratory tract infection – very common in the paediatric patient
 Nausea and vomiting (which generally precedes abdominal pain, as compared to the sequence in appendicitis)
Diagnosis: Primary = right-sided mesenteric lymphadenopathy that does not have an identifiable acute inflammatory process or demonstrates mild (<5 mm) wall thickening of the terminal ileum
Secondary = lymphadenopathy that is associated with a specific, identifiable intraabdominal inflammatory process e.g. Crohn’s disease, infectious ileitis), the mesenteric adenitis is considered secondary
Investigations:
FBC normally showing a leukocytosis (↑WCC)
Urinalysis to exclude UTI
Blood/Stool cultures if appropriate
CT scan with contrast to exclude appendicitis
Gastroenteritis
Definition: Gastroenteritis is a nonspecific term used to describe a condition in which there is a combination of nausea, vomiting, diarrhoea and abdominal pain. It is most commonly observed in the context of viral illnesses (40%) e.g. norovirus, rotavirus and adenovirus but equally bacterial organisms are commonly implicated e.g. Campylobacter spp. Escherichia coli, Salmonella, Shigella or toxins from Staphylococcus aureus, Bacillus cereus or Clostridium. Other parasitic pathogens Cryptosporidium,  Amoebiasis should also be considered. It is believed that 20% of the UK population develop a form of infectious gastroenteritis whereby often the causative agent is rarely isolated.
Risk factors:

 Poor personal hygiene and lack of sanitation increase the incidence
 Immunocompromised patients are vulnerable to gastroenteritis e.g. diabetics, HIV, oncology patients
 Long term antibiotic use
 Inappropriate food preparation, cooking and storing techniques leading to ingestion of enterotoxins
Features: Acute gastroenteritis is defined as diarrheal disease (≥3 episodes per day or at least 200 g of stool per day) of rapid onset that lasts <2 weeks and may be accompanied by nausea, vomiting, fever, or abdominal pain.
Signs: Low grade temperature, soft but mildly tender abdomen are often present +/- voluntary guarding. Signs of dehydration will often be present depending on the time course and severity of losses. Check for dry mucous membranes, tachycardia, hypotension, or altered mental status.
Gastroenteritis
Investigations
Stool investigations - microscopy (include ova, cysts and parasites), culture and sensitivity should be sent if any of the below apply:
         Presence of blood and/or mucus
         Immunocompromised patient
         Recent travel anywhere other than western Europe, North America, Australia or New Zealand
         Limited improvement by day seven of supportive management
Blood tests: FBCs, renal function and electrolytes to help guide management and supportive therapies
Red Flags: You should be able to identify patients who may need hospitalisation or evaluation for alternative causes of their presentation.
 Severe volume depletion/dehydration
 Abnormal electrolytes or renal function
 Bloody stool/rectal bleeding
 Weight loss
 Severe abdominal pain
 Prolonged symptoms (more than one week)
 Hospitalisation or antibiotic use in the past three to six months
 Age 65 or older
 Comorbidities (e.g. diabetes mellitus, immunocompromised)
 Pregnancy
Common Symptoms in Clinic –SBA Revision
From the brief description of symptoms reported in the below scenarios indicate the single most likely diagnosis:
A 44 year-old female with a history of Crohn's disease presents with central abdominal pain, vomiting and not opening her bowels for 48 hours – A 35 year-old female presents with colicky left iliac fossa pain. Her last period was 9 weeks ago and she reports a change in bowel habits - A 21 year old female university student comes to clinic 2 months before exams. She describes ongoing abdominal pain, bloating and a change in bowel habits indicating frequent diarrhoea with some mucus present but nil blood A 24 year-old male presents with pain in the right iliac fossa. He has not eaten for 24 hours and reports being nauseated in this time. On examination he has a low-grade temperature A 32 year old male has returned from a recent foreign business trip and reports 4 days of ongoing dull abdominal pain accompanied by multiple episodes of loose stools

Case Scenario 9
On review of the patient what is the most likely diagnosis?

Case scenario 9: Explanation
EXPLANATION:

The history, signs and symptoms are classical of acute appendicitis. She is in the high risk age bracket and the acute presentation should make you suspicious and query risk of perforation and rupture. She is reporting the classic triad of symptoms notably sharp pain at McBurney' s point

Note pregnancy testing should always be carried out on a female of child bearing age irrespective of sexual history and reported method of contraception. This can help guide you diagnosis and rule out pregnancy related complications and ensure appropriate scans are adopted. IN this case it is important to note CT scan would be the most specific for diagnosing appendicitis but pelvic/abdominal CT scans in pregnancy carry a small risk of fetal damage.

Vomiting and diarrhea may not always be present together however, either one combined with sharp abdomen pain points to appendicitis. Approximately 60% of patients will describe the pain as typically periumbilical to start with subsequent migration to the right lower quadrant as the inflammation progresses.

Her blood results indicate an acute inflammatory process evidenced by her elevated CRP and WCC. This along with her low grade pyrexia and her physical examination would produce a Alvarado score of at least 7 making appendicitis highly likely. There is clinical and biochemical evidence of dehydration which would require concurrent supportive management.
Case Scenario 10 – Wernicke’s Encephalopathy

Background :A 59-year-old gentleman who is known to consume large amounts of alcohol is admitted to the general medical ward with a four-week history of mild confusion, jaundice and abdominal distension. His partner who accompanies him states that his personality and conscious levels have been affected recently. He does not have ophthalmoplegia and his postural balance is not affected.

Examinations : On examination, he is able to communicate normally. He is noted to have musty aroma in his breath, scratch marks on his skin, multiple spider naevi, flapping tremor and an enlarged abdomen with distended veins.

Bloods Na: 145mmol/L, Ur: 6.5mmol/L, Cr: 120µmol/L, Albumin: 25g/L, ALT: 55iU/L (H), ALP: 80iU/L, GGT: 140u/L (H), Haemoglobin: 130g/L, MCV: 105fL (H), WCC: 12.0 (H)

Investigations: Free serum ammonia levels are found to be elevated. His thiamine level is normal.

Case Scenario 10
On review of the patient what is the most likely diagnosis?



Consider a provisional diagnosis and work your way through the content to help revise the above clinical conditions before confirming your answer.

Hepatocellular Carcinoma
Definition: Hepatocellular carcinoma (HCC) is a primary tumour of the liver and is the 5th most common cancer worldwide. It usually develops in the setting of chronic liver disease and cirrhosis particularly in patients with viral hepatitis chiefly chronic hepatitis B and C. HCC is frequently diagnosed late in its course because of the absence of pathognomonic symptoms and thus requires prompt investigation and management to aid prognosis. Tumour size and vascular invasion are indicative of poorer outcomes and require various modalities of imaging to detect this early. Overall, the median survival is 6 – 20 months as often at the time of diagnosis limited intervention is possible. Early detection is key as rapid and severe decompensation makes treatment futile.
Risk Factors
Liver cirrhosis secondary to:
 Hepatitis B/C
 Alcohol related cirrhosis with decompensation
 Haemochromatosis
 Primary Biliary Cirrhosis

Other risk factors include:
 Alpha-1 antitrypsin deficiency
 Glycogen storage disease
 Aflatoxin – metabolite of a fungus in groundnut
 Drugs: oral contraceptive pill, anabolic steroids
 Male (sex ratio varies 2-4:1)
Clinical features
Most common – rapid development of below highly suggestive of ΔHCC
 Weight loss and Anorexia
 Ascites
 Abdominal pain – especially RUQ

Less common:
 Obstructive jaundice caused by invasion of the biliary tree, compression of the intrahepatic duct, or rarely, as a result of haemobilia
 Diarrhoea
 Bone pain or dyspnea due to metastases
Hepatocellular Carcinoma
Investigations:
Alpha-fetoprotein: Commonly elevated in malignancy Serum levels greater than 500 mcg/L (normal range 10 and 20 mcg/L) in a high-risk patient is diagnostic of HCC. However, serum levels of AFP do not correlate well with other clinical features of HCC such as size, stage, or prognosis and is found to be normal in approx. 1/3 of patients with HCC.
Screening with ultrasound (+/- alpha-fetoprotein - AFP) should be considered for high risk groups such as:
      Patients liver cirrhosis secondary to hepatitis B & C or haemochromatosis
      Men with liver cirrhosis secondary to alcohol
US Imaging


CT Scan
See Right: Cirrhotic changes demonstrating a multifocal hepatoma (arrows) which is invading the portal vein (arrowhead). The spleen is enlarged due to portal hypertension secondary to cirrhosis.
Gilbert’s Syndrome Definition: Gilbert’s syndrome is a benign condition that has also been called "constitutional hepatic dysfunction" and "familial non-hemolytic jaundice“. This is manifests as a low threshold for individuals becoming jaundice secondary to defunct enzyme activity and reduced conjugation of bilirubin with glucuronic acid. Patients with Gilbert syndrome have a mutation in UGT1A1, the gene that encodes bilirubin-UGT which allows for the conversion of bilirubin into a water-soluble form that is readily excreted in bile. However, the normal liver architecture will be present and functionality is not affected. It is believed to affect up to 5% of the UK population and is commonly diagnosed as an incidental finding of slightly elevated bilirubin or brief asymptomatic episodes of jaundice. There is a male dominance of approx. 2:1.
Causes:
Due to the reduced activity of enzyme glucuronyl transferase which conjugates bilirubin and a few other lipophilic molecules. Conjugation renders the bilirubin water-soluble, after which is then excreted in bile into the duodenum. Therefore the build up of unconjugated bilirubin remains in the blood causing skin pigmentation changes = jaundice BUT will not alter the colour of urine.
Potential Triggers of Jaundice:
 Infection
 Dehydration
 Stress
 Loss of appetite/fasting
 Increased alcohol intake
 Over-exertion/exercise
 Menses
Gilbert’s Syndrome                  
                                                    

Clinical Features:
 Mostly asymptomatic as bilirubin levels remain too low to cause symptomatic jaundice
 Often picked up in adolescence with intercurrent illness displaying mild and episodic jaundice (scleral icterus)
 Nonspecific symptoms such as malaise, abdominal discomfort or fatigue often described
 It has been suggested some patients with Gilbert’s syndrome have a lower threshold for paracetamol overdose

Differential Diagnosis
While the syndrome is referred to as benign, in its early presentation it is clinically important to ensure other more severe diagnoses have been excluded. As jaundice and some of the accompanying non-specific symptoms can be a sign of haematological origin or liver pathology consider the below:
  Haemolysis – exclude with FBC, LDH and assess for elevated reticulocytes which would be elevated in a haemolytic anaemia
  Viral hepatitis – assess risk of exposure, family history. Perform specific antigen/antibody testing if any suspicion
  Cholestasis – can be exclude by the absence of LDH, low levels of conjugated bilirubin and US scan of bile ducts
Wernicke Encephalopathy Definition: Wernicke's encephalopathy is a neuropsychiatric disorder of an acute nature requiring immediate medical attention. This is most commonly seen in excess alcohol users and relates to thiamine (Vit B1) deficiency which can have various consequences on metabolic pathways and have lasting neurological complications known as Korsakoff Syndrome.
Rarer causes include: persistent vomiting, dietary deficiency stomach cancer,. A classic triad of ophthalmoplegia/nystagmus, ataxia and confusion may occur. In Wernicke's encephalopathy petechial haemorrhages occur in a variety of structures in the brain including the mamillary bodies and ventricle walls
Risk Factors:
 Chronic alcoholism
 Anorexia nervosa or dieting
 Hyperemesis of pregnancy
 Prolonged intravenous feeding without proper supplementation
 Prolonged fasting or starvation, or unbalanced nutrition, especially with refeeding
 Gastrointestinal surgery (especially bariatric surgery)
 Systemic malignancy
 Transplantation
 Haemodialysis or peritoneal dialysis
Wernicke Encephalopathy      
Clinical Features: It is rare for an individual to display all 3 of the classic signs whereby this has been documented in only 20% of cases. Altered mental state is the most common feature and other features include:

 Coma
 Hypotension
 Hypothermia
 Peripheral neuropathy – commonly of the lower limbs
Diagnosis: Neuroimaging and blood tests can be helpful to confirm a diagnosis or rule out differentials however, a diagnosis is made based on clinical features and patient history. A low threshold for treatment should be considered for known alcohol users. Early thiamine replacement with improving symptoms helps to confirm the diagnosis.
Most untreated patients proceed to develop a Korsakoff psychosis with loss of recent memory, disordered time appreciation, and confabulation. This is irreversible and therefore Wernicke’s should be treated rapidly and aggressively with parenteral and oral thiamine replacement.
Assessment and management of alcohol withdrawal should also be implemented – further information can be found at NICE guidance CG100
Budd-Chiari Syndrome Definition: Defined as hepatic venous outflow tract obstruction usually due to an underlying disease process commonly relating to a hypercoagulable state. Primary Budd-Chiari syndrome is present when there is obstruction due to a primarily venous process commonly thrombosis of the hepatic veins which includes the major vessels (right, middle and left hepatic vein). Occlusion can often also include the intrahepatic or suprahepatic inferior vena cava. The interruption of blood flow out of the liver cause patients to develop postsinusoidal portal hypertension, which leads to complications similar to those observed in patients with cirrhosis.  This is due to ongoing congestion causing architectural changes to the structure. Diagnosis should be considered for any patients presenting with acute liver failure, acute hepatitis, or chronic liver disease, particularly if the patient has known risk factors.
Risk Factors:

 Myeloproliferative diseases
 Malignancy – HCC is the most common
 Infection and benign lesions of liver
 Hypercoagulable states
 Behçet disease
 Membranous webs of the IVC or hepatic vein
 Idiopathic
Clinical features:
Acute- rapid clinical presentations (within a week)
 Severe RUQ pain
 Hepatomegaly
 Ascites and Jaundice may present later

Chronic symptoms develop over months
 Epigastric or RUQ discomfort
 Hepatomegaly
 Ascites
 Stigmata of chronic liver disease
Budd-Chiari Syndrome
Physical Examination

 Abdominal palpation – organomegaly
 Gross ascites +/- peripheral oedema
 Stigmata of liver disease e.g. spider naevus
 Jaundice
 Liver flap

Image courtesy of James Heilman, MD

Image courtesy of DermNetNZ.org
Diagnosis: Patients with acute liver failure and the presence of hepatomegaly, right upper quadrant pain, and ascites should increase the suspicion for Budd-Chiari syndrome. Noninvasive investigations e.g. Doppler ultrasonography are used to assess for thrombosis whilst also assessing the portal and splenic circulation to exclude concurrent portal or splenic vein thrombosis. CT, MRI or venography can be performed to confirm the diagnosis if there is uncertainty after US. Rarely liver biopsy may be indicated showing cirrhotic changes in the chronic presentation of the Budd-Chiari. Other tests may show mildly elevated ALP, ALT and bilirubin but is very variable.

Liver Anatomy Revision
Couinaud Classification: This divides the liver into 8 segments, each with its own vascular supply and biliary drainage. In the centre of each segment there is a branch of the portal vein, hepatic artery and bile duct. In the periphery of each segment there is vascular outflow through the hepatic veins. This is clinically significant as each segment acts independently and can technically be individually dissected without disturbing overall liver function.

Image courtesy of Polygon data is from BodyParts3D
Image courtesy of Polygon data is from BodyParts3D
Right hepatic vein  divides the right lobe into anterior and posterior segments.
Middle hepatic vein divides the liver into right and left lobes (or right and left hemiliver). This plane runs from the inferior vena cava to the gallbladder fossa.
The Falciform ligament divides the left lobe into a medial- segment 4 and a lateral part - segment 2 and 3.
The portal vein  divides the liver into upper and lower segments. The left and right portal veins branch superiorly and inferiorly to project into the centre of each segment.
Hepatic Encephalopathy
Definition: An often reversible impairment of neuropsychiatric function associated with impaired hepatic function due to subsequent metabolic complications. It is suggested that an increase in ammonia concentration is implicated which is neurotoxic. There may also be a role for inhibitory neurotransmission through gamma-aminobutyric acid (GABA) receptors in the central nervous system and changes in central neurotransmitters and circulating amino acids causing neuromuscular dysfunction. It can present on a spectrum ranging from subtle changes to mental state and memory to severe neuropsychiatric deficit such hepatic coma. Often features can also include neuromuscular changes/deficits and can be transient feature of the condition but equally indicative of the severity of the condition.

Risk Factors: HE is a well established complication of alcoholic liver disease and subsequent cirrhosis occuring in up to 50% of these patients. This is also a clinical sign of decompensated liver disease.

Common precipitants:
 Acute kidney injury.
 Electrolyte imbalance
 GI bleeding
 Infection
 Constipation
 Sedative drugs - eg, opiates, benzodiazepines, antidepressants and antipsychotic drugs.
 Diuretics
 High protein intake
Hepatic Encephalopathy
Encephalopathy Grading System
Hepatic encephalopathy can be graded using the Conn score (also called West Haven classification ) in which higher scores indicate a higher severity.
Hepatic Encephalopathy
Clinical Features: Often Hepatic Encephalopathy is divided into 2 main categories – covert at the lesser end of the spectrum and overt displaying more obvious clinical signs.

Covert HE :
 Normal mental status
 Transient/episodic confusion
 Episodic – normally occurring once in 6 months
 Irritability
 Disordered sleep
 Subtle cognitive decay vs baseline
 Nil asterixis on examination

Overt HE:
 Often lacks orientation
 Potential altered level of consciousness
 Cognitive dysfunction
 Sustained changes in behaviour
 Abnormal speech
 Ataxic gait and asterixis often present
 Neuromuscular dysfunction – rigidity, hyporeflexia
Diagnosis
History and Physical exam – mental state exam, cognitive function vs baseline, sleeping pattern, memory, work performance and ability to perform ADLs, assess gait, abdominal exam assessing for signs of liver disease, lower and upper limb examination assessing for neuromuscular impairment
Laboratory Tests – Serum ammonia (not diagnostic but may be elevated), rule out other causes e.g. electrolytes, glucose, urea and intoxication. Checking LFTs will help to determine underlying liver damage but may be normal
Imaging – Noncontrast head CT which may also show cerebral oedema, MRI if CT inconclusive
Special tests – psychometric tests assessing visual perception, visuospatial orientation, visual construction, motor speed and accuracy, concentration, attention, and memory
Case Scenario 10
On review of the patient what is the most likely diagnosis?

Case scenario 10: Explanation
EXPLANATION:

The history of alcohol abuse associated with the presence of musty aroma in his breath (fetor hepaticus - secondary to the exhalation of mercaptans), scratch marks, spider naevi, flapping tremor, impaired personality and change in conscious levels, and an enlarged abdomen with distended veins (porto-systemic anastamosis) is strongly suggestive of alcoholic liver disease leading to hepatic encephalopathy and associated decompensation.

Approximately 30% of patients with end-stage liver disease experience significant encephalopathy, approaching coma. An elevated arterial or free venous serum ammonia level is an important finding in patients with hepatic encephalopathy, although in about 10% of patients with hepatic encephalopathy the serum ammonium levels may be normal. (Ammonia, produced in the GI tract by bacterial degradation of amines, amino acids, purines and urea, is normally detoxified in the liver by conversion to urea by the Krebs-Henseleit cycle. However, in cirrhosis there is a decreased mass of functioning hepatocytes for detoxification of the ammonia and the porto-systemic shunting may divert blood containing ammonia away from the liver to the systemic circulation.)

HCC is a potential diagnosis as the patient fulfils much of the diagnostic criteria although the presence of neurological deficits makes this less likely. Gilbert’s syndrome is a benign condition impacting bilirubin metabolism and would present in a much younger patient. Wernicke encephalopathy and Korsakoff’s psychosis are caused due to thiamine (Vitamin B-1) deficiency, frequently seen in alcoholics (but can be found in disorders associated with malnutrition and in patients on long-term haemodialysis or with HIV). Wernicke encephalopathy is characterised by ataxia, confusion, ophthalmoplegia and impairment of short-term memory. The normal levels of serum thiamine and the absence of other symptoms of Wernicke encephalopathy and Korsakoff’s psychosis rules out this condition in this patient. The physical signs of Budd-Chiari are present in the scenario although the single best answer remains hepatic encephalopathy as this helps to fully link the underlying pathology with the reported presentation.